Google Groups no longer supports new Usenet posts or subscriptions. Historical content remains viewable.
Dismiss

What lead Einstein to the theory of special relativity

11 views
Skip to first unread message

Shane williams

unread,
Nov 26, 2010, 6:07:27 AM11/26/10
to
Can anyone explain what lead Einstein to the theory of special
relativity? I know it was something to do with Maxwell's equations.
I guess he had no way of testing or observing that time slows down the
faster you go so what lead him to that conclusion? Did it explain
something that had been observed but there was no explanation for?
How did he do it?

TIA

artful

unread,
Nov 26, 2010, 6:19:14 AM11/26/10
to

Have a look at his 1905 paper on it for a start.

Helmut Wabnig

unread,
Nov 26, 2010, 7:32:27 AM11/26/10
to

His wife, Mileva Maric.
http://en.wikipedia.org/wiki/Mileva_Mari%C4%87

w.

harald

unread,
Nov 26, 2010, 9:28:18 AM11/26/10
to

Einstein had studied Maxwell's equations in part from Lorentz's papers
due to lack of updated textbooks. These brought to his attention that
some small asymmetries were expected from motion, which however were
not observed. He was next excited by a paper of Poincare who publicly
asked for a theory in which the same laws of nature - incl. the laws
of Maxwell - are valid in all inertial reference systems, so that no
effect of inertial motion can be detected at all. Anyway, here's how
Einstein put it in his paper:

"It is known that Maxwell's electrodynamics--as usually understood at
the present time--when applied to moving bodies, leads to asymmetries
which do not appear to be inherent in the phenomena. Take, for
example, the reciprocal electrodynamic action of a magnet and a
conductor." Etc, see the introduction here:
http://www.fourmilab.ch/etexts/einstein/specrel/www/

Much discussed at that time, also in Lorentz's papers, was an
experiment with an interferometer by Michelson and Morley:
http://en.wikipedia.org/wiki/Michelson%E2%80%93Morley_experiment

The new theory based on Poincare's principle of relativity had just
been found by Lorentz and Poincare when Einstein wrote his paper, but
he pretended not to be aware of that. And concerning the slowdown of
clocks, that aspect of the theory was neglected in the earlier papers.
Einstein was the first to elaborate on that and to make predictions
about it. The logical need for this "time dilation" is perhaps easiest
to understand from the later experiment by Kennedy and Thorndike,
which is also mentioned near the end of the Wikipedia article here
above.

Regards,
Harald

PD

unread,
Nov 26, 2010, 2:31:23 PM11/26/10
to

Yes, it was primarily Maxwell's equations and a peculiarity about
them. They appeared to violate the principle that the laws of physics
should be the same regardless of inertial motion -- a principle
discovered by Galileo and imbued in all of Newton's mechanics.
However, this appearance of violation was with the assumption that
inertial motion corresponded with a transformation of coordinates like
what you see in freshman physics classes: all velocities change by the
v (the inertial motion), and spatial coordinates change by x'=x + vt
or something like that. It turns out that this assumption turns out to
be wrong, and Einstein used the principle of relativity to figure out
what the transformation had to be like for inertial motion.

This explained why no ballistic change in lightspeed had ever been
detected, such as in the Michelson-Morley experiment.

However, it made a number of other rather unusual predictions that had
not been born out by experiment yet. That is the inherent risk built
into proposing any new theory -- it makes bold predictions of things
not yet seen and therefore not yet validated. It took a number of
years for a significant number of those predictions to be borne out by
measurement.

Androcles

unread,
Nov 26, 2010, 2:36:16 PM11/26/10
to

"PD" <thedrap...@gmail.com> wrote in message
news:4338818c-0303-45e3...@m20g2000prc.googlegroups.com...

On Nov 26, 5:07 am, Shane williams <shane.2471...@gmail.com> wrote:
> Can anyone explain what lead Einstein to the theory of special
> relativity? I know it was something to do with Maxwell's equations.
> I guess he had no way of testing or observing that time slows down the
> faster you go so what lead him to that conclusion? Did it explain
> something that had been observed but there was no explanation for?
> How did he do it?
>
> TIA

Yes, it was primarily Maxwell's equations

====================================
Oh, so that's what "the unsuccessful attempts to discover any motion of the
earth relatively to the ``light medium,''" is all about.
(together with the unsuccessful attempts to travel backwards in time).


Androcles

unread,
Nov 26, 2010, 2:58:28 PM11/26/10
to

"Shane williams" <shane....@gmail.com> wrote in message
news:4d591507-72b1-4014...@c17g2000prm.googlegroups.com...
Quite easy, he said
<http://www.fourmilab.ch/etexts/einstein/specrel/www/figures/img11.gif>

time light leaves = 1:00:00 o'clock.
time light reflects = 1:45:00 o'clock.
time light returns = 2:00:00 o'clock.

duration of outbound trip = 45 minutes,
duration of inbound trip = 15 minutes.

Then he said "I'm the great Einstein, I'm god, I insist 45 = 15. We
establish by definition that the ``time'' required by light to travel from A
to B equals the ``time'' it requires to travel from B to A" because I
fuckin' say so and dammit, don't you dare argue with ME, I'm a fuckin'
genius!

Then he said 1/2 * (45 +15) = 45, (the other half is 15 but forget that),
and since the round trip time is 60 mins in the stationary frame and 45 mins
in the moving frame, time is dilated.

Bingo, that was how time dilation got invented.
You do believe in miracles, don't you?


Allen Esterson

unread,
Nov 26, 2010, 6:23:34 PM11/26/10
to
Helmut Wabnig wrote:
>His wife, Mileva Maric.
>http://en.wikipedia.org/wiki/Mileva_Mari%C4%87

The Wikipedia page cited here says this about the contention that
Mileva Maric assisted Einstein with the 1905 papers: "However, the
overwhelming consensus among professional historians of physics is
that she did not."

Mileva twice failed the Zurich Polytechnic teaching diploma exams, and
there is no sound evidence that she collaborated on any of Einstein's
groundbreaking work.


Tom Roberts

unread,
Nov 27, 2010, 11:25:33 AM11/27/10
to
Shane williams wrote:
> Can anyone explain what lead Einstein to the theory of special
> relativity?

Arguments of symmetry in the laws of physics, with Maxwell's equations
considered as a law of physics.

In the first paragraph of his 1905 paper, he pointed out that for a magnet and
conductor it is their relative motion that is all that matters, while Maxwell's
equations AS THEN UNDERSTOOD had an asymmetry, in that this depended on their
absolute velocities (relative to the aether). In later writings he mentioned
that he had also considered a thought experiment in which an observer traveled
with speed c along a light wave -- Maxwell's equations cannot be satisfied for
such an observer (the fields must be unmoving to such a co-moving observer, so
all time derivatives are zero in this frame, and thus the equations cannot be
satisfied while still being a wave in the stationary frame [@]).

[@] This is resolved in classical electrodynamics and SR, because
it is impossible for an observer to move with such speed.

His 1905 paper was based on his convictions that a) the Principle of Relativity
is a symmetry of the laws of nature, and b) Maxwell's equations are a law of
nature. The conclusion is that these two postulates [#] lead one to reject
Galilean invariance in favor of Poincaré invariance. Note that the difference
between them is unobservable in our everyday lives, and in most experiments up
to that time; today there are literally zillions of experiments that show the
validity of Poincaré invariance, and which refute Galilean invariance by
enormous factors.

Since then, the theory now known as Classical Electrodynamics
has been developed, in which Maxwell's equations are valid in
any inertial frame. Here "classical" means "pre quantum", not
"pre relativity". It is based directly on the work of Lorentz,
Einstein, and Poincaré in 1904-1910: Lorentz first displayed
the eponymous transforms in their modern form, and Poincaré
derived the full invariance group of Maxwell's equations, but it
was Einstein who described the physical consequences and brought
it together into a coherent theory with a sensible basis.

[#] In his paper the second of these is replaced by a postulate
about the speed of light. It was really (b) that was in his mind,
but he phrased it in terms of light for pedagogical reasons. There
are many choices for the second postulate, including none at all
(using experiments to select the valid transform equations from
among the 3 groups permitted by the PoR).

Einstein's 1905 paper has become one of the most important papers in the history
of science, not only because it introduced SR to the world, but more importantly
because it showed the importance and power of such symmetry arguments. Since
then, Poincaré invariance has become a pillar of modern physics, but it has been
reduced to local validity because of the lessons learned from General
Relativity. Today EVERY mainstream theory of physics obeys local Poincaré
invariance.

[Other responses have touched on this, but none have adequately
discussed the significance of symmetry principles, which is the
key point.]


Tom Roberts

Titus

unread,
Nov 27, 2010, 3:41:55 PM11/27/10
to
On 26 Nov., 12:07, Shane williams <shane.2471...@gmail.com> wrote:
> Can anyone explain what lead Einstein to the theory of special
> relativity?

See
http://en.wikipedia.org/wiki/History_of_special_relativity

Regards,

Shane williams

unread,
Dec 4, 2010, 9:20:05 PM12/4/10
to

Thanks.

So how much can a non-physicist (but with a science degree) expect to
understand about special relativity and the fact that time slows down
and length contracts the faster you go. If I get a book about special
relativity from the library, can I expect it to explain why we think
that time slows down.

It seems to me that to get any idea of why we would expect that time
slows down and the speed of light is the same for all observers, you
need to understand Maxwell's equations and the things that lead
Einstein to the theory of special relativity. Without understanding
that, all a book can do is explain what the postulates of special
relativity are and what the consequences of that are.

I have a long-standing desire to understand more about special
relativity but I'm thinking that unless I learn bout Maxwell's
equations and whatever lead Einstein to relativity, I'm not going to
understand why anyone would expect that time slows down the faster you
go, let alone how Einstein came up with E = MC*C

mpc755

unread,
Dec 4, 2010, 9:59:44 PM12/4/10
to

Time is a concept. The rate at which a clock ticks has nothing to do
with time.

If you own a battery operated clock and it begins to tick slower has
time changed, or do you replace the batteries?

The rate at which an atomic clock ticks is determined by the force of
the aether in which it exists.

Uncle Ben

unread,
Dec 4, 2010, 10:29:18 PM12/4/10
to
On Dec 4, 9:20 pm, Shane williams <shane.2471...@gmail.com> wrote:
> go, let alone how Einstein came up with E = MC*C- Hide quoted text -
>
> - Show quoted text -

Shane, none of us really understands relativity at an intuitive level,
but you can learn better than most what it says, at least.

To my mind one of the most difficult things to learn is exactly what
is meant by saying that speeding clocks run slow. It may not quite be
what you think.

What many miss at first is that clocks can run at different rates at
the same time. I am purposely being a bit challenging when I put it
this way, but I will explain.

One could say that the altitude of a building can have different
values simultaneously. This is because you can choose different
reference levels. The altude referred to the street is one thing, and
the altitude referred to mean sea level is another. There is clearly
no contradiction here.

Consider a clock on a train moving through a station. Special
relativity says that the clock can have different rates at the same
time: With respect to the train, the clock is motionless and has its
maximum rate. With respect to the station, the same clock is moving
and has a slower rate.

This is quite strange, but experiments show it to be true. I don't
believe that study of Maxwell's equations will help you much to
believe it until you reach the level of sophistication of a Tom
Roberts or Paul Draper (PD), but a review of the Hafely-Keating
experiment (see wikipedia) may be helpful. They flew around the world
with an atomic clock and saw that the clock gaiined or lost time
according to which way they travelled: with or against the earth's
rotation (east to west or west to east).

Subsequent refinements of this experiment have confirmed time dilation
to be quantitatively extremely close to what Einstein's theories
predict, including a second effect due to Earth's gravity.

Uncle Ben

Androcles

unread,
Dec 5, 2010, 1:12:32 AM12/5/10
to

"Shane williams" <shane....@gmail.com> wrote in message
news:18b34950-494f-4e28...@z17g2000prz.googlegroups.com...

Thanks.

===========================================
Take a course in mathematics and then read this afterwards.
<http://www.fourmilab.ch/etexts/einstein/specrel/www/>

It isn't difficult to understand bullshit without stepping in it,
or to understand insanity without being insane.
Here's a primer:
<http://www.androcles01.pwp.blueyonder.co.uk/SR4kids/SR4kids.htm>


--
"Let there be given a stationary rigid rod; and let its length be L as
measured by a measuring-rod which is also stationary. We now imagine
the axis of the rod lying along the axis of x of the stationary system of
co-ordinates, and that a uniform motion of parallel translation with
velocity v along the axis of x in the direction of increasing x is then
imparted to the rod. We now inquire as to the length of the moving rod" --
Einstein
"The length to be discovered by the operation (b) we will call ``the length
of the (moving) rod in the stationary system.''"-- Einstein

"This we shall determine on the basis of our two principles, and we shall
find that it differs from L." -- Einstein.

AND THE ANSWER IS...

"xi = (x-vt)/sqrt(1 - v^2/c^2)" -- Einstein.

Yep, xi differs from L, Greek letters differ from Roman letters.

In agreement with experience we further assume the deranged babbling
incompetent cretin couldn't answer his own inquiry, he was too stupid
to realise xi is greater than L when he wrote 'for v=c all moving
objects--viewed from the "stationary'' system--shrivel up into plane
figures', whereas his own equation shows they stretch to infinity...
sqrt(1-c^2/c^2) = 0.


"But the ray moves relatively to the initial point of k, when measured in
the stationary system, with the velocity c-v" - Einstein
"the velocity of light in our theory plays the part, physically, of an
infinitely great velocity" - Einstein.
"In agreement with experience we further assume the quantity
2AB/(t'A -tA) = c to be a universal constant--the velocity of light in
empty space." -- Einstein
He was right. The distance from A to A divided by the time it takes
to get there is undefined. Anyone that divides by zero is a lunatic.


--
--
Predictions of relativity.

"In agreement with experience we further assume the quantity
2AB/(t'A-tA) = c to be a universal constant--the velocity of
light in empty space." --Einstein

In agreement with bullshit:
"the velocity of light in our theory plays the part, physically,
of an infinitely great velocity" -- Einstein.


"We establish by definition that "the ``time'' required by light
to travel from A to B equals the ``time'' it requires to travel

from B to A."-- Einstein.


"In accordance with definition the two clocks synchronize if
tB-tA = t'A-tB", but tB-tA is 1/2(t'A-tA).

Hence tB-tA plays the part, physically, of half an infinitesimally
small duration of time.


Clock A can see the Earth and Earth can see clock A,no matter
how far apart they are they are synchronized, the light signals
between them play the part, physically, of taking half of an
infinitesimally small duration of time, which plays the part,
physically, of zero.


In agreement with experience:
Clock A reads 6:00 am at dawn, it's a perfect clock.
In agreement with experience:
Clock B reads 12:00 pm at noon, it's a perfect clock.


In agreement with bullshit:
"if one of two synchronous clocks at A is moved in a closed curve
with constant velocity until it returns to A, the journey lasting
t seconds, then by the clock which has remained at rest the travelled
clock on its arrival at A will be 1/2 tv^2/c^2 second slow."- Einstein.

In agreement with Einstein's assumption:
Clock A meets clock B at A and is 6 hours slow. Both clocks
synchronize with Earth, because "in accordance with definition
the two clocks synchronize if 0 = 0"-- Einstein.

In disagreement with the Principle of Simultaneity (A meets B when B
meets A):
Clock A meets clock B at dawn and clock B sees clock A arrive at noon.

In agreement with experience:
The dork Einstein plays the part, physically, of a deranged lying cretin.

Androcles

unread,
Dec 5, 2010, 1:29:01 AM12/5/10
to

"Uncle Ben" <b...@greenba.com> wrote in message
news:96055f3c-9545-4cab...@r29g2000yqj.googlegroups.com...

=================================================
None of *you* are sane, Bonehead.
Many of *us* understand it completely at all levels.

In *your* theory " the velocity of light in our theory plays the part,
physically, of an infinitely great velocity" --§ 4. Physical Meaning of
the Equations Obtained in Respect to Moving Rigid Bodies and Moving
Clocks -- ON THE ELECTRODYNAMICS OF MOVING BODIES By A. Einstein

But in *our* theory
"It seems that Light is propagated in time, spending in its passage from
the sun to us about seven Minutes of time:" -- DEFIN. II of Opticks Or,
A Treatise of the Reflections, Refractions, Inflections and Colours of
Light - Sir Isaac Newton.


To my mind one of the most difficult things to learn is exactly what
is meant by saying that speeding clocks run slow. It may not quite be
what you think.

===============================================
You don't have a mind, Bonehead.

What many miss at first is that clocks can run at different rates at
the same time. I am purposely being a bit challenging when I put it
this way, but I will explain.

================================================
Does "at the same time" mean "simultaneously", Bonehead? What
does "rate" mean, Bonehead? Begin by "explaining" your definitions.


One could say that the altitude of a building can have different
values simultaneously. This is because you can choose different
reference levels. The altude referred to the street is one thing, and
the altitude referred to mean sea level is another. There is clearly
no contradiction here.

=============================================
One could say that the altitude of a building or the speed of light
can have only one value simultaneously. This is because you can
choose different reference levels. The alTItude referred to the street
or the speed of light referred to empty space is one thing, and the
altitude referred to mean sea level or the speed of lighr referred to
a moving object is another. There is clearly no contradiction here,
until you say both have the same value, then the contradiction shows
your insanity.


Consider a clock on a train moving through a station. Special
relativity says that the clock can have different rates at the same
time: With respect to the train, the clock is motionless and has its
maximum rate. With respect to the station, the same clock is moving
and has a slower rate.

This is quite strange, but experiments show it to be true.

===============================================
This is quite impossible unless the clock is broken, and you are a
lying deranged shitheaded moron.


I don't believe
==================
Good, neither do I. Now shut the fuck up.

--
Bonehead (aka "I'm not sue") is grieving, he's in denial that the moving
xi is greater than x' and angry at Androcles for murdering his precious
"LT".
Poor Uncle Bonehead, can't his bonehead out of his arse.
Bwhahahahaha!

Message has been deleted

Shane williams

unread,
Dec 5, 2010, 3:36:36 AM12/5/10
to
On Dec 5, 4:29 pm, Uncle Ben <b...@greenba.com> wrote:
>
> Consider a clock on a train moving through a station.  Special
> relativity says that the clock can have different rates at the same
> time: With respect to the train, the clock is motionless and has its
> maximum rate. With respect to the station, the same clock is moving
> and has a slower rate.
>
> This is quite strange, but experiments show it to be true. I don't
> believe that study of Maxwell's equations will help you much to
> believe it until you reach the level of sophistication of a Tom
> Roberts or Paul Draper (PD), but a review of the Hafely-Keating
> experiment (see wikipedia) may be helpful. They flew around the world
> with an atomic clock and saw that the clock gaiined or lost time
> according to which way they travelled: with or against the earth's
> rotation (east to west or west to east).
>
> Subsequent refinements of this experiment have confirmed time dilation
> to be quantitatively extremely close to what Einstein's theories
> predict, including a second effect due to Earth's gravity.

Thanks.

I wish books would say that there's some underlying (complex) theory
that lead to SRT and that it wasn't just dreamed up and that it can't
be explained intuitively.

Is it possible to look at how an atomic clock works (I don't know how
it works) and ask why should it always tick at the same rate - like,
who would know why it ticks at a certain rate i.e. it's not
surprising that the rate it ticks at is affected by how fast it's
moving because who could possibly guess at how fast it should tick -
therefore, time dilation is neither surprising nor unsurprising - it's
just nature ??

I wonder why they didn't do the Hafele-keating experiment more times.
Of course I accept it but I would be more convinced if they did it 50
times and got the same results every time.


Androcles

unread,
Dec 5, 2010, 3:55:20 AM12/5/10
to

"photon" <graeme....@gmail.com> wrote in message
news:4a90574f-d5e6-4a0a...@o9g2000pre.googlegroups.com...

I wonder why they didn't do the Hafele-keating experiment more times.
Of course I accept it but I would be more convinced if they did it 50
times and got the same results every time.
===============================================
Hafele-Keating is done twice a day by 27 GPS satellites which orbit the
Earth
every 12 hours and no satellite has ever seen another satellite tick slower
than
any other satellite or itself. If one ever did it would be broken and not
much
good for GPS.
I wonder why YOU don't do the Hafele-Keating experiment more times.
Of course I do NOT accept it but I would be more convinced of your
insanity and broken clock if you did it 50 times and got the same results
every time.


Shane williams

unread,
Dec 5, 2010, 3:59:42 AM12/5/10
to
On Dec 5, 4:29 pm, Uncle Ben <b...@greenba.com> wrote:
>
> Shane, none of us really understands relativity at an intuitive level,
> but you can learn better than most what it says, at least.
>
> To my mind one of the most difficult things to learn is exactly what
> is meant by saying that speeding clocks run slow.  It may not quite be
> what you think.
>
> What many miss at first is that clocks can run at different rates at
> the same time.  I am purposely being a bit challenging when I put it
> this way, but I will explain.


Yeah, this is a bit I haven't quite got my head around. It seems to
me that clocks can't actually run at different rates at the same
time. For a person in a spaceship travelling at half the speed of
light, time ticks over "normally". He has no idea that his clock is
ticking more slowly than a clock somewhere else because he has no way
of comparing them. In the hafele-keating experiment, the clocks were
able to be compared when the plane stopped, but while the plane is
moving, there's no way to compare them and the person on the ground
has no way to tell what rate the moving clock is ticking at until the
plane stops. i.e. it's not meaningful to say that the clock on the
spaceship ticks over at a different rate as seen by the guy on the
spaceship, compared to what the guy on the ground sees ??

I wonder if the plane was to fly right past the person on the ground
one thousand times on its way round the world, and then at the exact
time that the plane passes the person on the ground, they both write
down the time they see on each other's clocks. What numbers would
they write down?

Androcles

unread,
Dec 5, 2010, 4:57:58 AM12/5/10
to

"Shane williams" <shane....@gmail.com> wrote in message
news:935c5db1-9ba7-4f71...@n32g2000pre.googlegroups.com...

On Dec 5, 4:29 pm, Uncle Ben <b...@greenba.com> wrote:
>
> Shane, none of us really understands relativity at an intuitive level,
> but you can learn better than most what it says, at least.
>
> To my mind one of the most difficult things to learn is exactly what
> is meant by saying that speeding clocks run slow. It may not quite be
> what you think.
>
> What many miss at first is that clocks can run at different rates at
> the same time. I am purposely being a bit challenging when I put it
> this way, but I will explain.


Yeah, this is a bit I haven't quite got my head around. It seems to
me that clocks can't actually run at different rates at the same
time. For a person in a spaceship travelling at half the speed of
light, time ticks over "normally". He has no idea that his clock is
ticking more slowly than a clock somewhere else because he has no way
of comparing them.

===================================================
Of course he does, you are either deliberately lying or extremely stupid.

Jerry

unread,
Dec 5, 2010, 5:49:36 AM12/5/10
to
On Dec 5, 2:36 am, Shane williams <shane.2471...@gmail.com> wrote:

> I wonder why they didn't do the Hafele-keating experiment more times.
> Of course I accept it but I would be more convinced if they did it 50
> times and got the same results every time.

The Hafele-Keating experiment, in its day, stretched the limits
of atomic clock technology. Indeed, the PUBLISHED graphs are, to
say the least, rather unconvincing. It was only after analysis
of the data using a quite sophisticated technique, "correlated
rate change analysis", that decent error bounds could be placed
on their results.

The reason why the H-K experiment has not been repeated is that
atomic clock technology has progressed to the point that
detection of relativistic effects is now COMPLETELY ROUTINE, such
that, for example, gravitational time dilation has been measured
by antique atomic clock hobbyists going on a weekend outing with
the kids:
http://leapsecond.com/great2005/tour/

Current state-of-the-art clocks can detect time dilation at the
speed of a person running a 400 meter dash, or gravitational red
shift from a height difference equivalent to a small step stool:
http://www.nist.gov/public_affairs/releases/aluminum-atomic-clock_092310.cfm

The existence of relativistic time dilation effects is no longer
a matter of experimental debate, but is instead, a routine
engineering concern:
http://tycho.usno.navy.mil/ptti/ptti2002/paper20.pdf

Jerry

Androcles

unread,
Dec 5, 2010, 6:23:50 AM12/5/10
to

"Jerry" <Cephalobu...@comcast.net> wrote in message
news:8a7582f3-8aa4-48eb...@k22g2000yqh.googlegroups.com...

Jerry
=======================================
Current state-of-the-art technology can make dinosaurs walk in Jurassic Park
and the author of Wikileaks guilty of rape - especially American freedom of
speech technology. Nasty little fucks like you will always lie to the tune
of
your masters.


Synchronize two vacuum enclosed identical horizontal light clocks
side-by-side and leave to run for 6 months in two identical chest
freezers (for environmental control). Note any relative drift.
<http://www.androcles01.pwp.blueyonder.co.uk/lightclock.gif>

Place one horizontal light clock at the top of the Burj Khalifa
<http://www.burjkhalifa.ae/>
and leave the other at the base. Leave to run for 6 months.
Bring the clocks together again, note any relative drift.

If the clocks DO read the same count (with drift allowed) then NIST
got it wrong, there was no time dilation due to altitude difference.
<http://www.nist.gov/public_affairs/releases/aluminum-atomic-clock_092310.cfm>

If the clocks do NOT read the same count (with drift allowed) due to
time dilation then NIST got it wrong, the speed of light cannot be a
universal constant.
<http://physics.nist.gov/cgi-bin/cuu/Value?c>

Either way, NIST are useless yankee wankers and WRONG.

Uncle Ben

unread,
Dec 5, 2010, 12:48:58 PM12/5/10
to

Shane, two things:

1. I should have said, A given clock can run at many different rates
at the same time. Not just two or more clocks, but the SAME clock.
Read the analogy again: The SAME building can have more than one
altitude ... etc.

2. With regard to that buzzy fly that kieeps annoying us, I have
several notes about him on my webpage www.greenba.com.

Uncle Ben

Androcles

unread,
Dec 5, 2010, 1:12:48 PM12/5/10
to

"Uncle Ben" <b...@greenba.com> wrote in message
news:c5496e03-6806-403c...@l20g2000vbd.googlegroups.com...

Shane, two things:

================================================
Bwahahahahahahahaha!
HAHAHAHAHAHAHAhahahahahaaaaaa....(wheeze) ...
hahahahahahahaha....

Stop it, Bonehead, my sides are splitting!
A given ruler can measure many different inches at the same centimetre!


Tom Roberts

unread,
Dec 5, 2010, 1:41:35 PM12/5/10
to
Uncle Ben wrote:
> 1. I should have said, A given clock can run at many different rates
> at the same time. Not just two or more clocks, but the SAME clock.

Such puns are not very helpful. When you claim "this clock runs at many
different rates at the same time", you implicitly use a pun on the word "rates",
but did not bother to tell your reader of this abuse of the language.

It is not the clock ITSELF that "runs at different rates", but rather other
observers who MEASURE its tick rate to be different. This is a HUGE difference,
and an important one. This is appears to be confusing the original poster, in
part because of your poorly worded statements. Your way of stating it is
confusing, and as far as I am concerned, just plain wrong -- the clock has a
single rate at which it ticks, and it always ticks at its usual rate [#];
observers moving relative to it will measure different values for its rate, due
to the different ways they measure it.

[#] After all, this is what we mean by "clock".

One must consider such "measurements" more generally than just literal
measurements. There are physical consequences for the variations in clock rates
relative to other frames, such as the ability to construct pion beam lines.


> Read the analogy again: The SAME building can have more than one
> altitude ... etc.

This pun on the word "altitude" is also confusing and not very useful. Say,
rather, "this building can be measured to have different altitudes relative to
different references". There is no point in hiding the essential difference you
are trying to convey.

In English, when we say "this object has property X", we implicitly
mean that X is an intrinsic property of the object, and does not
depend on extraneous conditions such as who is looking at the object.
Your wording violates this basic English principle. When we say "John
has a height of 6 feet", nobody has the slightest expectation that
an astronaut passing by at 0.5 c would measure 6 feet; that value is
what John himself measures. Similarly, the rate at which a clock ticks
is what IT ITSELF measures, which is always its usual rate.


Tom Roberts

Tom Roberts

unread,
Dec 5, 2010, 1:52:54 PM12/5/10
to
Shane williams wrote:
> Is it possible to look at how an atomic clock works (I don't know how
> it works) and ask why should it always tick at the same rate

Yes. An atomic clock uses the atomic structure of Cesium atoms to derive a
highly stable oscillator at 9,192,631,770 Hz. This oscillator is governed by the
laws of quantum mechanics. In SR, the laws of physics are the same in any
inertial frame, so in any inertial frame such a clock will tick at that rate,
regardless of which inertial frame is used.

Hence an atomic clock ticks at its usual rate, as long as it is at rest in an
inertial frame. This, of course, says nothing at all about how observers moving
relative to the clock might measure its rate, this only describes its tick rate
in its rest frame.


> - like,
> who would know why it ticks at a certain rate i.e. it's not
> surprising that the rate it ticks at is affected by how fast it's
> moving because who could possibly guess at how fast it should tick -
> therefore, time dilation is neither surprising nor unsurprising - it's
> just nature ??

You're trying to follow Uncle Ben's wording, which is both confusing and wrong.
Say, rather, that an atomic clock always ticks at 9,192,631,770 Hz regardless of
its inertial frame, but that observers moving relative to it will measure other
values.


> I wonder why they didn't do the Hafele-keating experiment more times.

It was very expensive to purchase the airline tickets, and very cumbersome to
manage the clocks. Other measurements quite similar to it have been performed.
And they only borrowed the atomic clocks for a limited time period.

BTW there are MUCH better measurements of "time dilation", and the "twin
paradox"; see the FAQ for references:

http://math.ucr.edu/home/baez/physics/Relativity/SR/experiments.html


Tom Roberts

Androcles

unread,
Dec 5, 2010, 1:55:48 PM12/5/10
to

"Tom Roberts" <tjrobe...@sbcglobal.net> wrote in message
news:--qdnd6f6Od...@giganews.com...

Yeah, you tell 'im, 'Umpty. The clock ticks at one second per second
and 6 foot John is 1 foot per foot.
Bwhahahahahahahhahahaha!

Tom Roberts

unread,
Dec 5, 2010, 2:18:49 PM12/5/10
to
Shane williams wrote:
> So how much can a non-physicist (but with a science degree) expect to
> understand about special relativity and the fact that time slows down
> and length contracts the faster you go.

One must be very careful in phrasing this. In SR, time does not really "slow
down", and objects do not really "contract" with motion. It is only MEASUREMENTS
of moving objects that are affected ("measurement" considered generally). That
is, NOTHING happens to the moving object, what has changed is the RELATIONSHIP
between the object and the frame used to observe it.

This is a direct consequence of the Principle of Relativity: the
laws of physics are the same in all inertial frames. So a clock
moving inertially will behave the same regardless of which inertial
frame it is at rest in, because the laws of physics governing its
operation are the same.

Here's an analogy: consider a ladder and a doorway. In some orientations the
ladder can be carried through the doorway, and in other orientations it cannot.
Does rotating the ladder change the ladder ITSELF in any way? - NO. What changes
is the relationship between ladder and doorway. In SR, both "time dilation" and
"length contraction" are modeled in a similar way, as rotations in the 4-d
spacetime, which change the relationship between the object and the frame used
for measurement. Yes, relative motion is a ROTATION in SR; unlike the ladder
rotating in the X-Y (horizontal) plane, an object moving along the X axis is
rotated in the X-T plane, which is something you never observe in your everyday
life because the effects are far too small; for relative speeds approaching c,
however, the effects can be enormous. "Length contraction" is the projection on
X of this rotation, and "time dilation" is the projection on T; projections on Y
and Z are unaffected by this relative motion along X.


> If I get a book about special
> relativity from the library, can I expect it to explain why we think
> that time slows down.

Yes. But a good one will not phrase it that way. Instead, it will discuss
MEASUREMENTS of moving clocks and how their tick rates are measured to be slower
than for identical clocks at rest.


> It seems to


> me that clocks can't actually run at different rates at the same
> time. For a person in a spaceship travelling at half the speed of
> light, time ticks over "normally". He has no idea that his clock is
> ticking more slowly than a clock somewhere else because he has no way
> of comparing them.

Right. A clock cannot possibly "tick at many different rates at the same time"
unless one abuses the language. This is a fundamental problem with the way Uncle
Ben is attempting to "explain" this. IMHO his approach fails miserably.

"Moving clocks run slow" is a sound bite that cannot possibly
capture the subtleties of SR. Indeed, it is so distorted that
it is downright wrong. Moving clocks tick at their usual rates,
but they are measured to have slower tick rates, due to the
nature of the measurement procedure one must use.


Tom Roberts

Androcles

unread,
Dec 5, 2010, 2:39:25 PM12/5/10
to

"Tom Roberts" <tjrobe...@sbcglobal.net> wrote in message
news:6amdncrKmvQ...@giganews.com...

Humpty Roberts can run his mouth but is totally ignorant of algebra.

--
"I don't know what you mean by 'relativistic,' "Alice said.
Humpty Roberts smiled contemptuously. "Of course you don't -- till I tell
you. I meant "there's a nice knock-down argument for you!'"
"But `relativistic' doesn't mean `a nice knock-down argument,'" Alice
objected.
"When I use a word," Humpty Roberts said in a rather a scornful tone, "it
means just what I choose it to mean -- neither more nor less.
"The question is," said Alice, "whether you can make words mean different
things."
"The question is," said Humpty Roberts, "which is to be master -- that's
all."
Alice was too much puzzled to say anything, so after a minute Humpty
Roberts began again.
"They've a temper, some of them -- particularly verbs, they're the
proudest -- adjectives you can do anything with, but not verbs -- however, I
can manage the whole lot! Impenetrability! That's what I say!"


--
r_AB/(c+v) = r_AB/(c-v). References given:
<http://www.fourmilab.ch/etexts/einstein/specrel/www/figures/img6.gif>
<http://www.fourmilab.ch/etexts/einstein/specrel/www/figures/img11.gif>

Let r_AB = 480 million metres,
let c = 300 million metres/sec,
let v = 180 million metres/sec.

480/(300-180) = 480/(300 +180)
480/(120) = 480/(480)
4 = 1

"Humpty Roberts in Relativityland" - by Albert Einstein of Ulm.


Uncle Ben

unread,
Dec 5, 2010, 5:50:27 PM12/5/10
to
>   , when we say> Tom Roberts

With respect, I disagree.

You say, "In English, when we say "this object has property X", we
implicitly mean that X is an intrinsic property of the object ... ." I
want to shake this inferential habit.

I want the student to keep vividly in mind that the rate of a clock
depends not only on the clock but also on the frame of reference that
is pertinent to the discussion. Given that infinitely many frames of
reference can be constructed in which to regard a clock, so are there
many rates it can have. No observer is required, only a discusant.

This is not psychology; this is physics.

Otherwise we lay the groundwork for other misunderstandings, namely
that the phenomena of relativity are mere appearances and not
reality. We introduce an anthropomorpic element that is actually
irrelevant. It is like insisting that when a tree falls in the
forest, there is no sound. That may be true for the psychologist, but
not for the physicist.

My statement that you say is wrong is wrong only under the weight of
the implication you assume. In my usage, it is correct, and the habit
of thought I recommend to the student will prevent him from falling
into the trap in which kenseto, Henry Wilson, and noeinstein are
imprisoned.

The way I state the facts may seem quixotic, but it intended to shock
the student -- to force the student's attention violently to the
assertion of relativity of time and space as a matter of physics, not
psychology.

Uncle Ben

Androcles

unread,
Dec 5, 2010, 6:22:19 PM12/5/10
to

"Uncle Ben" <b...@greenba.com> wrote in message
news:430771f4-0645-45ad...@c17g2000prm.googlegroups.com...

With respect, I disagree.

Uncle Ben
===========================================
With disrespect, I don't agree. You are caught in the trap of
senile dementia, trying to steal some time and give it to length,
then stupid enough to call it time "dilation" and length "contraction"
when it is really time contraction and length dilation.
Moving clocks run slow (haha) and moving rods stretch (if you
are deranged, which you are).

--
Test of GR.

Wendy Parker

unread,
Dec 5, 2010, 6:56:29 PM12/5/10
to
Androcles wrote:
> Yeah, you tell 'im, 'Umpty. The clock ticks at one second per second
> and 6 foot John is 1 foot per foot.
> Bwhahahahahahahhahahaha!

Shut the fuck up, stupid heap of shit.

Tom Roberts

unread,
Dec 5, 2010, 8:32:11 PM12/5/10
to
Tom Roberts wrote:
> An atomic clock uses the atomic structure of Cesium atoms to derive
> a highly stable oscillator at 9,192,631,770 Hz.

I forgot to mention that other atoms can be used in atomic clocks, notably
hydrogen and rubidium; they have quite different frequencies. I described the
atomic clocks used in the ISO definition of the second.


Tom Roberts

Daryl McCullough

unread,
Dec 5, 2010, 8:37:45 PM12/5/10
to
Uncle Ben says...

>The way I state the facts may seem quixotic, but it intended to shock
>the student -- to force the student's attention violently to the
>assertion of relativity of time and space as a matter of physics, not
>psychology.

I don't understand the distinction you are trying to make. What is
an example of a way of describing relativity that would make it seem
that it is a matter of psychology?

--
Daryl McCullough
Ithaca, NY

Tom Roberts

unread,
Dec 5, 2010, 8:47:00 PM12/5/10
to
Uncle Ben wrote:
> On Dec 5, 1:41 pm, Tom Roberts <tjroberts...@sbcglobal.net> wrote:
> You say, "In English, when we say "this object has property X", we
> implicitly mean that X is an intrinsic property of the object ... ." I
> want to shake this inferential habit.

You cannot possibly do that. This is how the language is used. Live with it. You
have no choice -- you cannot unilaterally change the meanings of English words
and phrases; especially not the way other people read and interpret your words.


> I want the student to keep vividly in mind that the rate of a clock
> depends not only on the clock but also on the frame of reference that
> is pertinent to the discussion.

BUT IT DOESN'T. As I said, the rate of a clock is FIXED, because that's what we
mean by "clock". Only by abusing the language can you make that statement. Don't
do that!


> This is not psychology; this is physics.

No. This is you trying to change how people use the English language, an attempt
doomed to failure because the usual interpretation of such phrases is so very
useful in zillions of other contexts.


> Otherwise we lay the groundwork for other misunderstandings, namely
> that the phenomena of relativity are mere appearances and not
> reality.

Only when one is careless in wording. Yes, one must be careful, because
relativity is full of subtleties, and the language has evolved independent of
relativity. So no sound bites can capture the essence of SR, one must include a
rather long explanation, because the words do not "line up" with the actual
effects of SR. Live with it; you have no choice.

Why do you want to hide the essential point of your discussion? State the
reference when buildings "have multiple altitudes". State the frames used when
clocks "have multiple rates". Do that consistently and you'll find you have no
need to abuse the language and use such error-prone, hidden puns.

Your phrasing is VERY MUCH WORSE than the possibility of implying that these
effects are "mere appearance". Because your phrasing is DEFINITELY WRONG.


Tom Roberts

Uncle Ben

unread,
Dec 5, 2010, 9:01:12 PM12/5/10
to

I agree that I am using the language a little differently than
convention dictates. But you are too!

You emphasize that the varying rates or lengths are only MEASURED, not
intrinsic. No wondner the student's first reaction will be, "well, why
don't they measure it differently if the usual measurement is not
correct?"

There is no-one that we know of in distant galaxies doing any
measurement. Yet we presume that relativistic effects occur there as
well as here.

To put it another way, we all accept the uncertainty principle in QM.
Measurement does affect what is being measured. The way you put
things is that there is a similar principle at work in relativity.
Measurement cannot be relied upon to give the truth.

I say that you are changing the meaning of "measurement" when you
distinguish between what is measured and what is true. I prefer to
say that physicists making measurements are discovering what is true:
that the lifetime of a cosmic-ray muon IS what is it MEASURED to be.
And if the answer depends on its speed w.r.t. the laboratory, get used
to it!

Ben

Uncle Ben

unread,
Dec 5, 2010, 10:51:07 PM12/5/10
to

Daryl, the most egregious example is the persistent reference to
"observers", as if human perception is required. Tom's term is
"measured", as if someone has to measure (observe) a quantity for the
dilation or contraction to occur.

There are physicists who seriously contend that quantum effects depend
on observation by a conscious mind, which IMHO is laughable.

Students often interpret the requirement of an observer to imply that
relativity effects are illusions, or that the transit time of light to
the observer explains it all. Anything to escape the puzzling truth!

Tom wants to distinguish between what is observed and what is true, as
I read him. I find it more definite to assert that observation is a
means of learning the truth.

This debate is connected to the one over the reality of relativistic
effects, in which you and I and Harald took one side and Tom and Tim
Shuba took the other, IIRC.

Androcles

unread,
Dec 6, 2010, 2:31:36 AM12/6/10
to

"Uncle Ben" <b...@greenba.com> wrote in message
news:fe4bbbc7-53cc-42c2...@j32g2000prh.googlegroups.com...

Ben

==============================================
Oh dear... to agree with Bonehead is anathema, but now I have to. It's about
time you stood up to that dishonourable cretin Humpty Roberts, deer Uncle
Bonehead, and had the courage to drop that "with respect" pretentiousness.
Respect has to be earned and mathematics has to be proven, without the
psychobabble. Do that and you'll have respect.

Let r_AB = 480 million metres,
let c = 300 million metres/sec,
let v = 180 million metres/sec.

480/(300-180) = 480/(300 +180)
480/(120) = 480/(480)
4 = 1

"In agreement with experience we further assume" four seconds "play the
part, physically, of" one second.

Einstein psychobabble!


Shane williams

unread,
Dec 6, 2010, 7:14:51 AM12/6/10
to

ok. It seems that if you do take an atomic clock on a journey and
bring it back to where it started, that it would read a different time
to a clock that didn't go on the journey, even if they were both
always at the same height so that gravitation didn't change, so I'm a
little bit confused about why you say it ticks at its usual rate -
but no more confused than before! The article on time dilation on
Wikipedia suggests that humans could theoretically go on an ultra high
speed journey and come back to earth billions of (earth) years later.

Anyway, thanks to you and Uncle Ben for taking the trouble to try and
explain things. I'm going to read the wikibook on Wikipedia about
SR. It looks quite good.

Message has been deleted

Daryl McCullough

unread,
Dec 6, 2010, 7:30:28 AM12/6/10
to
Uncle Ben says...

>
>On Dec 5, 8:37=A0pm, stevendaryl3...@yahoo.com (Daryl McCullough) wrote:

>> I don't understand the distinction you are trying to make. What is
>> an example of a way of describing relativity that would make it seem
>> that it is a matter of psychology?

>Daryl, the most egregious example is the persistent reference to


>"observers", as if human perception is required. Tom's term is
>"measured", as if someone has to measure (observe) a quantity for the
>dilation or contraction to occur.

I think that these are kind of strange objections. Science is all
about *observing* and *measuring* and coming up with theories to
explain and predict our observations and our measurements.

But as for time dilation and length contraction, these effects require
*some* conventions of measurements or coordinate systems in order to
even be stated. They are coordinate-dependent effects. I don't see
any difference between "coordinate-dependent" and "measurement-dependent";
a coordinate system is a way to systematize collections of measurements.

>There are physicists who seriously contend that quantum effects depend
>on observation by a conscious mind, which IMHO is laughable.

In practice, this contention turns out to be a *practical* approach
to getting on with quantum physics without the need to get bogged
down in philosophy. You compute the evolution using the Schrodinger
equation up until the moment of observation, and then you use the
hypothesis of collapse to an eigenstate of the observable following
an observation. This recipe works, empirically, even if it isn't
satisfying, philosophically.

>Students often interpret the requirement of an observer to imply that
>relativity effects are illusions, or that the transit time of light to
>the observer explains it all. Anything to escape the puzzling truth!

Well, it seems to me that that's a golden opportunity to apply the
scientific method. A student proposes a hypothesis: that relativistic
effects are illusions due to the transit time for light. Then you
can propose an experiment (thought experiment) to test this hypothesis.
For example, the twin "paradox" thought experiment, in which one twin
zips off at 90% the speed of light for 10 years and returns, while
another twin remains at home the whole time. Ask what the "illusion"
theory would predict for their relative ages. Then show the actual
prediction of SR.

>Tom wants to distinguish between what is observed and what is true, as
>I read him. I find it more definite to assert that observation is a
>means of learning the truth.

I don't really see the distinction you're making. I'm sure Tom would
agree with that.

>This debate is connected to the one over the reality of relativistic
>effects, in which you and I and Harald took one side and Tom and Tim
>Shuba took the other, IIRC.

I argue both sides of that particular debate.

Uncle Ben

unread,
Dec 6, 2010, 7:51:29 AM12/6/10
to
> SR.  It looks quite good.- Hide quoted text -
>
> - Show quoted text -

Shane, it helps to be more careful how you say things in relativity.
Tom and I and almost everybody with a Ph.D. agree that a good clock
always ticks at its normal rate WHEN MEASURED IN A FRAME OF REFERENCE
IN WHICH IT IS AT REST. We say that this is its "proper time" and is
intrinsic to the clock.

The whole point of relativity is that what you measure depends on your
frame of reference. Before relativity, the frame of reference did not
matter.

The quibble that Tom and I disagree over is whether to stop right
there or to add that when you measure the rate of the same clock WITH
RESPECT TO A DIFFERENT FRAME OF REFERENCE, one can say that the clock
has a different rate. Tom says that that cannot be called a rate of
the clock, since it is not intrinsic to the clock, while I say that
that is a different rate of the clock.

It only matters when you want to say that relativistic effects are
"real" or are "merely" epiphenomena of your choice of frame of
reference. It is a philosophical point which may be just a matter of
taste.

Good luck with your study.

And ignore the kooks who still deny the truth of relativity in spite
of a hundred years of experimental confirmation.

Uncle Ben

Daryl McCullough

unread,
Dec 6, 2010, 8:02:49 AM12/6/10
to
Uncle Ben says...

>
>On Dec 5, 8:47=A0pm, Tom Roberts <tjroberts...@sbcglobal.net> wrote:

>I agree that I am using the language a little differently than
>convention dictates. But you are too!
>
>You emphasize that the varying rates or lengths are only MEASURED, not

>intrinsic. No wonder the student's first reaction will be, "well, why


>don't they measure it differently if the usual measurement is not
>correct?"

I think it is a mistake to think "Oh, if I had only used phrase X" or
"Oh, if I had only not used phrase Y" then the student would not have
become confused. Students become confused because it is a difficult
subject.

I think it is an important realization for a student to make that
the concept of the length of a moving object is an ill-defined
quantity until you operationalize it. What do we *mean* by "length"?
That's an *extremely* important question in relativity.

Your approach is to hide the word "measure" and act as if "length"
has an objective meaning. I think that's hiding information from
the student. Now, that might be appropriate or not, depending on
how you are teaching the subject. But eventually, the question of
how "length" is to be measured will come up (if not in Special
Relativity, then in General Relativity, where there really is
no objective notion of length).

>There is no-one that we know of in distant galaxies doing any
>measurement. Yet we presume that relativistic effects occur there as
>well as here.

The real answer to that question is to describe relativistic
effects in terms of coordinate-independent quantities. There are
no coordinate systems in distant galaxies, either.

But to speak in terms of measurement is to do a hypothetical:
if one were to do X, then Y would be observed. You don't have
to actually do X in order for the claim to make sense.

>To put it another way, we all accept the uncertainty principle in QM.
>Measurement does affect what is being measured. The way you put
>things is that there is a similar principle at work in relativity.
>Measurement cannot be relied upon to give the truth.

Well, a thorough discussion of measurements would explain the
relationship between measurements and the intrinsic properties
that give rise to measurements.

>I say that you are changing the meaning of "measurement" when you
>distinguish between what is measured and what is true.

No, it's not. The important thing to keep in mind is that some
aspects of a description in terms of measurement are conventional,
and some are intrinsic.

If I have a door that is 30 inches wide, and I have a box that
is 34 inches wide, can the box fit through the door? Very possibly
yes, if you turn it sideways. "Width" is not an intrinsic property
of a box; it depends on its orientation, as well.

>I prefer to say that physicists making measurements are discovering
>what is true: that the lifetime of a cosmic-ray muon IS what is it
>MEASURED to be.

If two different people measure the lifetime of the *same*
particle, and get two different results, then how can they both
be discovering what is true? The answer is that what is being
measured is not a lifetime of the particle, but a kind of
projection of the lifetime onto a particular axis, in the same
way that "width" is the projection of an object's extent onto
a particular axis.

If you always describe your measurements in coordinate-independent
terms then everyone will agree about the answers. If you use
coordinate-dependent terms, then different people will get different
answers, which usually means that it's not objective fact.

--
Daryl McCullough
Ithaca, NY

>And if the answer depends on its speed w.r.t. the laboratory, get used
>to it!
>
>Ben

Shane williams

unread,
Dec 6, 2010, 8:21:57 AM12/6/10
to


Hmm, I got Tom's comment about that earlier but I don't understand why
you're saying it to me now.


> Tom and I and almost everybody with a Ph.D. agree that a good clock
> always ticks at its normal rate WHEN MEASURED IN A FRAME OF REFERENCE
> IN WHICH IT IS AT REST. We say that this is its "proper time" and is
> intrinsic to the clock.

Sure, but you didn't dispute my statement that when a clock goes on a
journey and comes back, it reads a different time than when it stays
home, therefore, if the "usual rate" is the rate it would have ticked
over if it stayed at "home", then it didn't tick over at its usual
rate. According to wikipedia, the hafele keating experiment clock
actually ran fast. So maybe this is just an issue of what is meant by
"usual rate".


>
> The whole point of relativity is that what you measure depends on your
> frame of reference. Before relativity, the frame of reference did not
> matter.

Yeah, but it seems to be more than that because you can make clocks
run slow or fast by taking them on a trip.


>
> The quibble that Tom and I disagree over is whether to stop right
> there or to add that when you measure the rate of the same clock WITH
> RESPECT TO A DIFFERENT FRAME OF REFERENCE, one can say that the clock
> has a different rate.  Tom says that that cannot be called a rate of
> the clock, since it is not intrinsic to the clock, while I say that
> that is a different rate of the clock.
>
> It only matters when you want to say that relativistic effects are
> "real" or are "merely" epiphenomena of your choice of frame of
> reference.  It is a philosophical point which may be just a matter of
> taste.
>
> Good luck with your study.
>
> And ignore the kooks who still deny the truth of relativity in spite
> of a hundred years of experimental confirmation.

Yeah, I didn't read Androcles posts after I saw the abuse so I don't
know what his point of view is but he sure messed up this thread.

Thanks.

Androcles

unread,
Dec 6, 2010, 8:24:36 AM12/6/10
to

"Uncle Ben" <b...@greenba.com> wrote in message
news:5301d3fb-6bc4-4fa0...@k38g2000vbc.googlegroups.com...

Uncle Ben
=========================================

Bonehead (aka "I'm not sue") is grieving, he's in denial that the moving
xi is greater than x' and angry at Androcles for murdering his precious
"LT".
Poor Uncle Bonehead, can't his bonehead out of his arse.

It would be a miracle if Bonehead could produce a one microsecond of his
"experimental confirmation" of length contraction, the LYING
old git.

Bwhahahahaha!

Daryl McCullough

unread,
Dec 6, 2010, 8:31:39 AM12/6/10
to
Shane williams says...

>ok. It seems that if you do take an atomic clock on a journey and
>bring it back to where it started, that it would read a different time
>to a clock that didn't go on the journey, even if they were both
>always at the same height so that gravitation didn't change, so I'm a
>little bit confused about why you say it ticks at its usual rate -
>but no more confused than before!

Here's an analogy (some people hate analogies, but I think they
are essential for understanding things that you don't have direct
experience with): I have two roads that both connect Chicago to
Buffalo, NY. Each road has roadmarkers all along its
length. For the first road, there are 450 roadmarkers between
Buffalo and Chicago. For the second road, there are 680 roadmarkers
between Buffalo and Chicago. Is it correct to conclude that the
roadmarkers are spaced closer together for the second road than
for the first road?

No, the first road goes straight west from Buffalo to Chicago
for a total distance of 450 miles, while the second road goes
southwest for part of its length, then turns northwest for the
last part of its length, for a total distance of 680 miles.

The distance between cities depends on the path taken. Similarly,
the elapsed time between events in relativity depends on the path
taken.

Dean Elliot

unread,
Dec 6, 2010, 8:57:02 AM12/6/10
to
Daryl McCullough wrote:
> Uncle Ben says...

>>
>> Tom wants to distinguish between what is observed and what is true,
>> as I read him. I find it more definite to assert that observation
>> is a means of learning the truth.
>
> I don't really see the distinction you're making. I'm sure Tom would
> agree with that.

It seems as though Tom does distinguish between the Truth, which is beyond
the scope of physics,
and consistancy with a model, which is the result of the application of the
scientific method.

>
>> This debate is connected to the one over the reality of relativistic
>> effects, in which you and I and Harald took one side and Tom and Tim
>> Shuba took the other, IIRC.
>
> I argue both sides of that particular debate.

I agree that relativistic effects are projections of the hyperbolic symmetry
that result in the laws of conservation.


Androcles

unread,
Dec 6, 2010, 9:00:51 AM12/6/10
to

"Dean Elliot" <pi.r.cub...@gmail.com> wrote in message
news:vY5Lo.980$7V3...@newsfe21.iad...

Let r_AB = 480 million metres,
let c = 300 million metres/sec,
let v = 180 million metres/sec.

480/(300-180) = 480/(300 +180)
480/(120) = 480/(480)
4 = 1
"In agreement with experience we further assume" four seconds "play the
part, physically, of" one second.

Spamming Dean Elliot agrees with Einstein psychobabble!

Uncle Ben

unread,
Dec 6, 2010, 10:37:24 AM12/6/10
to

I certainly agree with the operational approach in the philosophy of
science, but once the philosophical issues are settled, I see no
problem with simply using the results without continually
perseverating on their philosophical basis.

The continued emphasis on what is observed, as if it were not real, is
what tripped up many, many physicists at the top of the profession
when confronted with Bell's spaceship problem in the 1960's. It was
not until the problem was recast in terms of a comoving frame of
reference at the breaking point that they were dragged into grumbling
submission.

It should have been clear to these physicists that the same phenomenon
should be describable from any inertial frame of reference, such as
the launch frame. In that frame it is undeniable that what is a
Lorentz contraction when the string is free turns into a state of
tension when its length is constrained.

In my conversations with well-known physicists at the time, it was
their insistance on the difference between observation and truth that
tripped them up.

Ben


Ben

Daryl McCullough

unread,
Dec 6, 2010, 11:20:56 AM12/6/10
to
Uncle Ben says...

>I certainly agree with the operational approach in the philosophy of
>science, but once the philosophical issues are settled, I see no
>problem with simply using the results without continually
>perseverating on their philosophical basis.

I don't know what you are considering to be an example of
"perseverating" on the philosophical basis of results. If
you are talking about the language of SR in terms of
"measurements", there is certainly an alternative to that,
which is the coordinate-free geometric approach, which doesn't
mention measurements. But in this approach, there is no "time
dilation" or "length contraction"---those are inherently
connected to particular coordinates and measurements. Of
course, any measurement can be described in a coordinate-free
manner, so you haven't lost any expressive power.

>The continued emphasis on what is observed, as if it were not real,
>is what tripped up many, many physicists at the top of the
>profession when confronted with Bell's spaceship problem in the
>1960's.

I would not say that that had anything to do with not treating
*measurements* as if they were real. Measurements tell you that
if two rockets follow identical acceleration patterns, then the
distance between the rockets will not change. The problem with
the glib conclusion, "therefore, the string will not break" is
about not thinking through what are the laws governing the
equilibrium length of a string.

>It was not until the problem was recast in terms of a comoving
>frame of reference at the breaking point that they were dragged
>into grumbling submission.

In other words, when they actually analyzed the problem, then
they got the right answer. You think that it's an important
consideration to be able to give a correct answer to a question
*without* thinking about it in detail? It's impressive when you
can do that but is it really important?

PD

unread,
Dec 6, 2010, 11:52:10 AM12/6/10
to
On Dec 5, 7:47 pm, Tom Roberts <tjroberts...@sbcglobal.net> wrote:
> Uncle Ben wrote:
> > On Dec 5, 1:41 pm, Tom Roberts <tjroberts...@sbcglobal.net> wrote:
> > You say, "In English, when we say "this object has property X", we
> > implicitly mean that X is an intrinsic property of the object ... ." I
> > want to shake this inferential habit.
>
> You cannot possibly do that. This is how the language is used. Live with it. You
> have no choice -- you cannot unilaterally change the meanings of English words
> and phrases; especially not the way other people read and interpret your words.

Actually, though, I agree with Uncle Ben -- as you already know.

The difficulty that novices face is that they *conflate* "intrinsic
properties" with "measured properties", as though it is the business
of science to measure intrinsic properties. And further, they will
claim that if a measurement does NOT yield an intrinsic property, then
what has happened is some sort of failure on the part of the
measurement, that it has been "fooled" to measure something "apparent"
and not "real".

This is CERTAINLY the demonstrated problem with Seto and Ralph
Rabbidge (Henry Wilson), who have been quite open about their stance
on this.

Where students benefit is from having this challenged right from the
start, so that they learn to break this habit. This can be done using
ordinary examples. A simple case is asking about the "reality" of a
shape of a trajectory in ordinary, 2D projectile motion under the
influence of gravity. Is the trajectory "really" straight or is it
"really" a parabola? For beginners, this is not a special case, and
they will readily jump on a claim that the shape of the trajectory,
since it is measured, must be intrinsically one or the other; and
furthermore, if that shape varies from frame to frame, then this is an
*apparent* effect and not on that reflects reality. It takes a little
doing to show them that, no, the shape of the trajectory just is frame-
dependent, and there is nothing wrong, imprecise, or veiled in a
measurement that shows it one way in one frame and another way in
another frame.

A similar statement can be made about *classical* velocity, where
beginners think that if you can measure it, then it must be an
intrinsic property, but it doesn't take much to provoke the
realization that it cannot be.

This realization, that there are some properties that are frame-
dependent, is one of the first things that good students stumble on in
their *first* year.

shuba

unread,
Dec 6, 2010, 3:35:46 PM12/6/10
to
PD wrote:

> This realization, that there are some properties that are frame-
> dependent, is one of the first things that good students stumble on in
> their *first* year.

Sure, and futher reflection leads to an understanding of why properties
considered intrinsic to a system are either fundamentally invariant or
*defined* in particular frame. The electron mass is .5MeV. If you measure
a particle with different energy, it doesn't mean it's not an electron.
First you need to apply the proper transformation to get the rest frame
energy.

Similarly, the stretching of a string is *defined* in its center-of mass
frame. In the Bell problem, it takes three lines of algebra to transform
and compare the space coordinates of the ends of the string into a
comoving rest frame where it's imediate clear that the string is
continually stretched with time. Yet people like Uncle Ben insist on
ignoring the definition and analyzing the problem in a complicated
manner, abandoning special relativity.


---Tim Shuba---

PD

unread,
Dec 6, 2010, 4:16:46 PM12/6/10
to
On Dec 5, 2:36 am, Shane williams <shane.2471...@gmail.com> wrote:
> On Dec 5, 4:29 pm, Uncle Ben <b...@greenba.com> wrote:
>
>
>
>
>
> > Consider a clock on a train moving through a station.  Special
> > relativity says that the clock can have different rates at the same
> > time: With respect to the train, the clock is motionless and has its
> > maximum rate. With respect to the station, the same clock is moving
> > and has a slower rate.
>
> > This is quite strange, but experiments show it to be true. I don't
> > believe that study of Maxwell's equations will help you much to
> > believe it until you reach the level of sophistication of a Tom
> > Roberts or Paul Draper (PD), but a review of the Hafely-Keating
> > experiment (see wikipedia) may be helpful. They flew around the world
> > with an atomic clock and saw that the clock gaiined or lost time
> > according to which way they travelled: with or against the earth's
> > rotation (east to west or west to east).
>
> > Subsequent refinements of this experiment have confirmed time dilation
> > to be quantitatively extremely close to what Einstein's theories
> > predict, including a second effect due to Earth's gravity.
>
> Thanks.
>
> I wish books would say that there's some underlying (complex) theory
> that lead to SRT and that it wasn't just dreamed up and that it can't
> be explained intuitively.

Ironically, I think it CAN be explained intuitively, with an appeal to
symmetry and simplicity to overcome the familiar. But the problem is,
novices sometimes prefer to hang on to the familiar even if it is more
complicated.

As an example, consider that in Newtonian physics, space and time are
completely independent, and that one (space) allows transformations
that don't involve time whatsoever. In relativity, these two disparate
things are combined into ONE thing, which is simpler. Similarly,
energy and momentum can be combined into one thing that some have
advocated be called "momenergy", where it becomes much more
intuitively obvious why we have two conservation laws, one dealing
with a 3 vector and another dealing with a scalar -- and in the
bargain we get a lovely invariant meaning of mass. Likewise, electric
and magnetic fields get combined into ONE electromagnetic thing. And
so on and so one.

>
> Is it possible to look at how an atomic clock works (I don't know how
> it works) and ask why should it always tick at the same rate  - like,
> who would know why it ticks at a certain rate  i.e. it's not
> surprising that the rate it ticks at is affected by how fast it's
> moving because who could possibly guess at how fast it should tick  -
> therefore, time dilation is neither surprising nor unsurprising - it's
> just nature  ??

Well, there's a couple things going on here.

First of all, there's the implicit trust that the laws of physics
don't care about the motion of a system. This is what is behind the
statement that the laws of physics governing the drop of a watermelon
are the same whether dropped from the top of a building or dropped
from a twin-prop airplane. The measured quantities might differ but
the laws of physics should remain the same.

Secondly, the observed slowing is precisely what is predicted by
relativity, and moreover the predicted slowing also hits it on the
nail for a number of other completely unrelated examples, including
the decays of subatomic particles which have nothing to do with atomic
clocks. It is highly unlikely that multiple processes of completely
different basis would just *happen* to accidentally slow by precisely
the amount predicted by relativity. One can argue it differently: If a
theory predicts that you will see a phenomenon in ANY of the cases
that are housed in a brown paper bag, one does not need to
systematically go through the entire contents of the brown paper bag
to verify that the prediction is correct -- you only have to pick one
or two at *random* to convince yourself it's correct.

>
> I wonder why they didn't do the Hafele-keating experiment more times.
> Of course I accept it but I would be more convinced if they did it 50
> times and got the same results every time.

PD

unread,
Dec 6, 2010, 4:20:06 PM12/6/10
to
On Dec 6, 2:35 pm, shuba <tim.sh...@lycos.ScPoAmM> wrote:
> PD wrote:
> > This realization, that there are some properties that are frame-
> > dependent, is one of the first things that good students stumble on in
> > their *first* year.
>
> Sure, and futher reflection leads to an understanding of why properties
> considered intrinsic to a system are either fundamentally invariant or
> *defined* in particular frame. The electron mass is .5MeV. If you measure
> a particle with different energy, it doesn't mean it's not an electron.
> First you need to apply the proper transformation to get the rest frame
> energy.

Yes, if it's the REST frame value that you're going after, agreed.
The electron's *rest* mass or *invariant* mass is 0.511 MeV.

PD

unread,
Dec 6, 2010, 4:26:11 PM12/6/10
to
On Dec 4, 8:20 pm, Shane williams <shane.2471...@gmail.com> wrote:

> On Nov 28, 5:25 am, Tom Roberts <tjroberts...@sbcglobal.net> wrote:
>
>
>
> > Shane williams wrote:
> > > Can anyone explain what lead Einstein to the theory of special
> > > relativity?
>
> > Arguments of symmetry in the laws of physics, with Maxwell's equations
> > considered as a law of physics.
>
> > In the first paragraph of his 1905 paper, he pointed out that for a magnet and
> > conductor it is their relative motion that is all that matters, while Maxwell's
> > equations AS THEN UNDERSTOOD had an asymmetry, in that this depended on their
> > absolute velocities (relative to the aether). In later writings he mentioned
> > that he had also considered a thought experiment in which an observer traveled
> > with speed c along a light wave -- Maxwell's equations cannot be satisfied for
> > such an observer (the fields must be unmoving to such a co-moving observer, so
> > all time derivatives are zero in this frame, and thus the equations cannot be
> > satisfied while still being a wave in the stationary frame [@]).
>
> >         [@] This is resolved in classical electrodynamics and SR, because
> >         it is impossible for an observer to move with such speed.
>
> > His 1905 paper was based on his convictions that a) the Principle of Relativity
> > is a symmetry of the laws of nature, and b) Maxwell's equations are a law of
> > nature. The conclusion is that these two postulates [#] lead one to reject
> > Galilean invariance in favor of Poincaré invariance. Note that the difference
> > between them is unobservable in our everyday lives, and in most experiments up
> > to that time; today there are literally zillions of experiments that show the
> > validity of Poincaré invariance, and which refute Galilean invariance by
> > enormous factors.
>
> >         Since then, the theory now known as Classical Electrodynamics
> >         has been developed, in which Maxwell's equations are valid in
> >         any inertial frame. Here "classical" means "pre quantum", not
> >         "pre relativity". It is based directly on the work of Lorentz,
> >         Einstein, and Poincaré in 1904-1910: Lorentz first displayed
> >         the eponymous transforms in their modern form, and Poincaré
> >         derived the full invariance group of Maxwell's equations, but it
> >         was Einstein who described the physical consequences and brought
> >         it together into a coherent theory with a sensible basis.
>
> >         [#] In his paper the second of these is replaced by a postulate
> >         about the speed of light. It was really (b) that was in his mind,
> >         but he phrased it in terms of light for pedagogical reasons. There
> >         are many choices for the second postulate, including none at all
> >         (using experiments to select the valid transform equations from
> >         among the 3 groups permitted by the PoR).
>
> > Einstein's 1905 paper has become one of the most important papers in the history
> > of science, not only because it introduced SR to the world, but more importantly
> > because it showed the importance and power of such symmetry arguments. Since
> > then, Poincaré invariance has become a pillar of modern physics, but it has been
> > reduced to local validity because of the lessons learned from General
> > Relativity. Today EVERY mainstream theory of physics obeys local Poincaré
> > invariance.
>
> >         [Other responses have touched on this, but none have adequately
> >          discussed the significance of symmetry principles, which is the
> >          key point.]
>
> > Tom Roberts
>
> Thanks.

>
> So how much can a non-physicist (but with a science degree) expect to
> understand about special relativity and the fact that time slows down
> and length contracts the faster you go.  If I get a book about special

> relativity from the library, can I expect it to explain why we think
> that time slows down.

You will probably be disappointed. Not because the answer isn't there,
but because you won't likely see the phrases "time slows down" or
"distance gets shorter". These are shallow catch-phrases that are cute
but don't really convey what's going on.

Instead, it helps to listen to a guide when the guide emphasizes the
question, "What does length of an object MEAN?" Therein you learn that
simultaneity becomes an essential ingredient to even what length
means. For example, you would not measure the length of a passing
train by marking the spot on the ground where the front of the train
passes you, and then walking toward the back of the train and marking
the spot on the ground where the back of the train passes you a short
time later. When you understand that simultaneity is central even to
the meaning of length, then it becomes less of a surprise that length
*so defined* is frame-dependent, because simultaneity is frame-
dependent (which you learn by a separate argument).

>
> It seems to me that to get any idea of why we would expect that time
> slows down and the speed of light is the same for all observers, you
> need to understand Maxwell's equations and the things that lead
> Einstein to the theory of special relativity.  Without understanding
> that, all a book can do is explain what the postulates of special
> relativity are and what the consequences of that are.
>
> I have a long-standing desire to understand more about special
> relativity but I'm thinking that unless I learn bout Maxwell's
> equations and whatever lead Einstein to relativity, I'm not going to
> understand why anyone would expect that time slows down the faster you
> go, let alone how Einstein came up with E = MC*C

Androcles

unread,
Dec 6, 2010, 4:34:39 PM12/6/10
to

"PD" <thedrap...@gmail.com> wrote in message
news:62f50b6b-7495-496a...@z17g2000prz.googlegroups.com...

============================================
Why not? Einstein did!

<http://www.androcles01.pwp.blueyonder.co.uk/Smart/LT.gif>
a.. The back of the train reaches the start of the fence when the light is
turned on.

b.. The light travels the length of the fence, 80 units.

c.. The time for light to travel 80 units is 16 ms. Speed of light is thus
80/16 = 5 units/ms.

d.. The light travels the length of the train, 40 units.

e.. The time for light to travel 40 units is 8 ms. Speed of light is thus
40/8 = 5 units/ms.

f.. The light travels back along the fence 20 units.

g.. The time for light to travel 20 units is 4 ms. Speed of light is thus
20/4 = 5 units/ms.

h.. The light travels back along the train 40 units.

i.. The time for light to travel 40 units is 8 ms. Speed of light is thus
40/8 = 5 units/ms.


--
No math, just more verbal diarrhea from a sociopathic bully.
"c = 1 and unitless in natural units." -- Phuckwit Duck
Ref:
a8760953-e947-4a3c...@w2g2000yqb.go
"(x1-x2)^2 + (y1-y2)^2 + (z1-z2)^2 - (t1-t2)^2 is invariant" -- Mallard.
"It turns out that you can verify curvature of a space without
ever stepping away from the space to see it embedded in a
higher dimension." - Mallard.
"Requests for *proof* will be routinely ignored in science because
theories are not proven in science."-- Mallard.
Algebra and irrational numbers have you fucked, Mallard.
If you do not comprehend it, it is only by virtue of your choice NOT
TO TRY, not to expend the effort.
You are a fucked up blind duck.
The rancid fat, Mal-lard, has nothing better to do with his time and
my computer is too dumb to care.
It is a lazy man who whines in the way you do, Mallard.
"You can't even keep track of the lies you say." -- Mallard
[sitting in the duck blind, waiting with a shotgun for a duck to appear]


Bruce Richmond

unread,
Dec 6, 2010, 5:58:04 PM12/6/10
to
On Dec 6, 8:02 am, stevendaryl3...@yahoo.com (Daryl McCullough) wrote:
> Uncle Ben says...
>
>
>
> >On Dec 5, 8:47=A0pm, Tom Roberts <tjroberts...@sbcglobal.net> wrote:
> >I agree that I am using the language a little differently than
> >convention dictates.  But you are too!
>
> >You emphasize that the varying rates or lengths are only MEASURED, not
> >intrinsic. No wonder the student's first reaction will be, "well, why
> >don't they measure it differently if the usual measurement is not
> >correct?"
>
> I think it is a mistake to think "Oh, if I had only used phrase X" or
> "Oh, if I had only not used phrase Y" then the student would not have
> become confused. Students become confused because it is a difficult
> subject.

It is not as difficult as some make it. The phrasing used by Ben
confuses the issue. A standard clock ticks at a rate of one second
per second in it's rest frame. It is measured to tick at different
rates in frames moving relative to it. Phrases such as "moving clocks
run slow" imply that the clock is somehow different, when in fact you
measure it to run slow because you are using a different coordinate
system in which the clocks making the measurement are synchronized
differently. The key to understanding SR is to keep in mind that each
frame uses its own coordinate system to make measurements, and because
of how those coordinate systems are set up you must use the Lorentz
transformation to translate between the coordinate systems.

> >Ben- Hide quoted text -

Androcles

unread,
Dec 6, 2010, 6:13:43 PM12/6/10
to

"Bruce Richmond" <bsr...@my-deja.com> wrote in message
news:9603b0eb-ff7e-4f4e...@e16g2000pri.googlegroups.com...

On Dec 6, 8:02 am, stevendaryl3...@yahoo.com (Daryl McCullough) wrote:
> Uncle Ben says...
>
>
>
> >On Dec 5, 8:47=A0pm, Tom Roberts <tjroberts...@sbcglobal.net> wrote:
> >I agree that I am using the language a little differently than
> >convention dictates. But you are too!
>
> >You emphasize that the varying rates or lengths are only MEASURED, not
> >intrinsic. No wonder the student's first reaction will be, "well, why
> >don't they measure it differently if the usual measurement is not
> >correct?"
>
> I think it is a mistake to think "Oh, if I had only used phrase X" or
> "Oh, if I had only not used phrase Y" then the student would not have
> become confused. Students become confused because it is a difficult
> subject.

It is not as difficult as some make it. The phrasing used by Ben
confuses the issue. A standard clock ticks at a rate of one second
per second in it's rest frame.

==========================================
Bwahahahahahahaha!
A standard ruler measures one inch per inch, a standard bathroom scale
measures one lb per lb, a standard house smells at one house per house,
a standard apartment laughs at one apartment per apartment, a standard
fuckwit babbles at one fuckwit per fuckwit, and you are a babbling fuckwit.
No matter how fast I drive, I ALWAYS travel one mile per mile and
one hour per hour, you ranting moron.

artful

unread,
Dec 6, 2010, 8:08:07 PM12/6/10
to
On Dec 7, 9:58 am, Bruce Richmond <bsr3...@my-deja.com> wrote:
> It is not as difficult as some make it.  The phrasing used by Ben
> confuses the issue.  A standard clock ticks at a rate of one second
> per second in it's rest frame.  It is measured to tick at different
> rates in frames moving relative to it.  Phrases such as "moving clocks
> run slow" imply that the clock is somehow different, when in fact you
> measure it to run slow because you are using a different coordinate
> system in which the clocks making the measurement are synchronized
> differently.  The key to understanding SR is to keep in mind that each
> frame uses its own coordinate system to make measurements, and because
> of how those coordinate systems are set up you must use the Lorentz
> transformation to translate between the coordinate systems.

Nicely said.

It is probably even better to say a moving clock is calculated as
ticking slower from measurements made in the observer's frame.

The observer cannot simply directly take a single measurement of a
moving clock's ticking rate. The observer has to take (at least) two
readings of the moving clock at different times (and because it is
moving, that means two different places) and compare those readings
with the difference in his own synchronized at-rest clocks at those
locations. From that he can calculate what the ticking rate is. SR
predicts what the result of those calculations would be.


artful

unread,
Dec 6, 2010, 8:15:16 PM12/6/10
to
On Dec 7, 10:13 am, "Androcles" <Headmas...@Hogwarts.physics_ac>
wrote:
> "Bruce Richmond" <bsr3...@my-deja.com> wrote in message

>
> news:9603b0eb-ff7e-4f4e...@e16g2000pri.googlegroups.com...
> On Dec 6, 8:02 am, stevendaryl3...@yahoo.com (Daryl McCullough) wrote:
>
>
>
>
>
>
>
>
>
> > Uncle Ben says...
>
> > >On Dec 5, 8:47=A0pm, Tom Roberts <tjroberts...@sbcglobal.net> wrote:
> > >I agree that I am using the language a little differently than
> > >convention dictates. But you are too!
>
> > >You emphasize that the varying rates or lengths are only MEASURED, not
> > >intrinsic. No wonder the student's first reaction will be, "well, why
> > >don't they measure it differently if the usual measurement is not
> > >correct?"
>
> > I think it is a mistake to think "Oh, if I had only used phrase X" or
> > "Oh, if I had only not used phrase Y" then the student would not have
> > become confused. Students become confused because it is a difficult
> > subject.
>
> It is not as difficult as some make it.  The phrasing used by Ben
> confuses the issue.  A standard clock ticks at a rate of one second
> per second in it's rest frame.
> ==========================================
> Bwahahahahahahaha!
> A standard ruler measures one inch per inch, a standard bathroom scale
> measures one lb per lb,

Exactly right .. otherwise they wouldn't be standard ruler/scale/
whatever. I'm not sure why you think that is funny.

A standard ruler will measure one inch per inch in its own frame no
matter how fast other observers are travelling past it .. the ruler
itself doesn't get "contracted" .. however, the MEASUREMENT of the
length of the ruler, made by a moving observer, is less

Uncle Ben

unread,
Dec 6, 2010, 8:15:31 PM12/6/10
to

Au contraire, mon ami.

Uncle Ben first exploits the comoving frame to persuade the student
that the string breaks.

Having made that lemonade, he squeezes yet more juice from the lemon.
He reconsiders the problem w.r.t. the launch frame to discover a new
theorem:

There exist relativistic stresses in constrained solids subjected to
acceleration (in addition to the classically understood compression
that propagates the acceleration).

Consider the alternative history that might have been if relativity
had preceded the development of magnetism.
It would be discovered that parallel beams of electrons repel each
other less strongly the faster the beam velocity. This might be
understood by transformation from
the comoving frame and observing time dilation.

Returning to the laboratory frame, one would discover magnetism as a
force partially cancelling the electrostatic repulsion of the beams.
Voila'! A useful new theory.

Uncle Ben

Androcles

unread,
Dec 6, 2010, 8:19:55 PM12/6/10
to

"Uncle Ben" <b...@greenba.com> wrote in message
news:3761e371-34d2-490b...@p8g2000vbs.googlegroups.com...

On Dec 6, 3:35 pm, shuba <tim.sh...@lycos.ScPoAmM> wrote:
> PD wrote:
> > This realization, that there are some properties that are frame-
> > dependent, is one of the first things that good students stumble on in
> > their *first* year.
>
> Sure, and futher reflection leads to an understanding of why properties
> considered intrinsic to a system are either fundamentally invariant or
> *defined* in particular frame. The electron mass is .5MeV. If you measure
> a particle with different energy, it doesn't mean it's not an electron.
> First you need to apply the proper transformation to get the rest frame
> energy.
>
> Similarly, the stretching of a string is *defined* in its center-of mass
> frame. In the Bell problem, it takes three lines of algebra to transform
> and compare the space coordinates of the ends of the string into a
> comoving rest frame where it's imediate clear that the string is
> continually stretched with time. Yet people like Uncle Ben insist on
> ignoring the definition and analyzing the problem in a complicated
> manner, abandoning special relativity.
>
> ---Tim Shuba---

Au contraire, mon ami.

Uncle Ben first exploits the comoving frame to persuade the student
that the string breaks.

======================================
Bwahahahahahahahaha!

Yeah, persuasion is much better than proof...
Fuckwit!

jem

unread,
Dec 7, 2010, 9:15:58 AM12/7/10
to
Bruce Richmond wrote:
> On Dec 6, 8:02 am, stevendaryl3...@yahoo.com (Daryl McCullough) wrote:
>> Uncle Ben says...
>>
>>
>>
>>> On Dec 5, 8:47=A0pm, Tom Roberts<tjroberts...@sbcglobal.net> wrote:
>>> I agree that I am using the language a little differently than
>>> convention dictates. But you are too!
>>
>>> You emphasize that the varying rates or lengths are only MEASURED, not
>>> intrinsic. No wonder the student's first reaction will be, "well, why
>>> don't they measure it differently if the usual measurement is not
>>> correct?"
>>
>> I think it is a mistake to think "Oh, if I had only used phrase X" or
>> "Oh, if I had only not used phrase Y" then the student would not have
>> become confused. Students become confused because it is a difficult
>> subject.
>
> It is not as difficult as some make it. The phrasing used by Ben
> confuses the issue. A standard clock ticks at a rate of one second
> per second in it's rest frame.

No. A standard clock doesn't tick at the rate of 1 second per second,
which is meaningless. It ticks at the rate of 1 second per *tick*
(more generally, X seconds per tick). IOW, for each specific
recognizable, repetitive change (i.e. tick) in a standard clock, the
clock registers 1 second (more generally, X seconds).

It is measured to tick at different
> rates in frames moving relative to it.

No. From any perspective whatsoever, the measurement of a clock's
tick rate (i.e., the amount of time the clock registers between each
tick it makes), will yield the same result.

Phrases such as "moving clocks
> run slow" imply that the clock is somehow different, when in fact you
> measure it to run slow

No. Only those who don't know how to measure a clock's tick rate will
measure a standard clock to run slow (or fast).

because you are using a different coordinate
> system in which the clocks making the measurement are synchronized
> differently.

No. It's because you're not measuring a clock's tick rate when you
determine the amount of time that elapses on *your* clock per tick of
some other clock (anymore than you'd be measuring the rate of someone
else's odometer by determining the ratio of the distance you travel
per tick of their odometer).

> The key to understanding SR is to keep in mind that each
> frame uses its own coordinate system to make measurements, and because
> of how those coordinate systems are set up you must use the Lorentz
> transformation to translate between the coordinate systems.
>

One key to understanding SR is to avoid thinking that you know the key
to understanding it, before you've understood it.


General comment:

The motivation here isn't pedantic stubbornness - the failure to
*emphasize* the distinction between "a clock's tick rate" and "a rate
at which a clock ticks", is a proven pedagogical faux pas that's
responsible for many deep-seated misconceptions, chief among which is
the failure to understand that, in the Einsteinian model, time
differences between events are a consequence of route-dependent time,
not differentially ticking clocks.

Tom Roberts

unread,
Dec 7, 2010, 10:06:34 AM12/7/10
to
Shane williams wrote:
> ok. It seems that if you do take an atomic clock on a journey and
> bring it back to where it started, that it would read a different time
> to a clock that didn't go on the journey, even if they were both
> always at the same height so that gravitation didn't change, so I'm a
> little bit confused about why you say it ticks at its usual rate

Let me stipulate that the clock that didn't go on the journey is at rest in an
inertial frame (any inertial frame). Call that frame S.

When measured in S, the moving clock has a tick rate slower than that of an
identical clock at rest in S. Is that the "rate of the clock" -- NO, it is "the
rate of the moving clock measured in S", which is a different quantity (with a
different value).

In Newtonian mechanics one did not need to be so careful in
wording; in relativity it is necessary.

This is no different from when we say "John is 6 feet tall" -- we mean what John
himself measures, and not what some astronaut speeding by might measure. So the
tick rate of a clock is its usual rate -- this is what these words mean.
Different results apply when one specifies a different frame in which the rate
is measured, and the difference is due to the difference in measuring procedures.

As I have said before, one must consider "measured" in general, and not just
when a human makes direct, quantitative measurements. For instance, this permits
us to construct pion beamlines.

Charged pions are unstable particles with an intrinsic lifetime
of 26 nanoseconds. In a beamline they move with speed >0.99 c.
If in the lab their lifetime was 26 ns then most of them would
decay within 7.8 meters. At Fermilab and CERN there are pion
beamlines hundreds of meters long, and only a few percent of the
pions decay in that distance. These beams have energy >100 GeV,
so in the lab the (intrinsic) pion lifetime is multiplied by a
factor of 714 or more, making the beamline length be a small
fraction of the decay distance of their pions.


Tom Roberts

kenseto

unread,
Dec 7, 2010, 10:49:09 AM12/7/10
to
On Dec 5, 3:36 am, Shane williams <shane.2471...@gmail.com> wrote:
> On Dec 5, 4:29 pm, Uncle Ben <b...@greenba.com> wrote:
>
>
>
>
>
>
>
> > Consider a clock on a train moving through a station.  Special
> > relativity says that the clock can have different rates at the same
> > time: With respect to the train, the clock is motionless and has its
> > maximum rate. With respect to the station, the same clock is moving
> > and has a slower rate.
>
> > This is quite strange, but experiments show it to be true. I don't
> > believe that study of Maxwell's equations will help you much to
> > believe it until you reach the level of sophistication of a Tom
> > Roberts or Paul Draper (PD), but a review of the Hafely-Keating
> > experiment (see wikipedia) may be helpful. They flew around the world
> > with an atomic clock and saw that the clock gaiined or lost time
> > according to which way they travelled: with or against the earth's
> > rotation (east to west or west to east).
>
> > Subsequent refinements of this experiment have confirmed time dilation
> > to be quantitatively extremely close to what Einstein's theories
> > predict, including a second effect due to Earth's gravity.
>
> Thanks.
>
> I wish books would say that there's some underlying (complex) theory
> that lead to SRT and that it wasn't just dreamed up and that it can't
> be explained intuitively.
>
> Is it possible to look at how an atomic clock works (I don't know how
> it works) and ask why should it always tick at the same rate  - like,
> who would know why it ticks at a certain rate  i.e. it's not
> surprising that the rate it ticks at is affected by how fast it's
> moving because who could possibly guess at how fast it should tick  -
> therefore, time dilation is neither surprising nor unsurprising - it's
> just nature  ??

Clocks run fast or slow is not due to time dilation. It is due to a
clock second contains a different amount of absolute time in different
frames (different states of absolute motion). The higher is the state
of absolute motion of a clcok the larger amount of absolute time is
contained by its clock second. For example the GPS clock second is
redefined to have 4.25 more periods of Cs 133 radiation. This
redefinition of the GPS second is designed to make the GPS clock
second contains the saem amount of absolute time as the ground clock
second....this makes the GPS clock in synch with the ground clock
permanently.

ken Seto

>
> I wonder why they didn't do the Hafele-keating experiment more times.
> Of course I accept it but I would be more convinced if they did it 50

> times and got the same results every time.- Hide quoted text -

kenseto

unread,
Dec 7, 2010, 5:23:45 PM12/7/10
to
On Dec 7, 10:06 am, Tom Roberts <tjroberts...@sbcglobal.net> wrote:
> Shane williams wrote:
> > ok.  It seems that if you do take an atomic clock on a journey and
> > bring it back to where it started, that it would read a different time
> > to a clock that didn't go on the journey, even if they were both
> > always at the same height so that gravitation didn't change, so I'm a
> > little bit confused about why you say it ticks at its usual rate
>
> Let me stipulate that the clock that didn't go on the journey is at rest in an
> inertial frame (any inertial frame). Call that frame S.
>
> When measured in S, the moving clock has a tick rate slower than that of an
> identical clock at rest in S. Is that the "rate of the clock" -- NO, it is "the
> rate of the moving clock measured in S", which is a different quantity (with a
> different value).
>
>         In Newtonian mechanics one did not need to be so careful in
>         wording; in relativity it is necessary.
>
> This is no different from when we say "John is 6 feet tall" -- we mean what John
> himself measures, and not what some astronaut speeding by might measure. So the
> tick rate of a clock is its usual rate -- this is what these words mean.
> Different results apply when one specifies a different frame in which the rate
> is measured, and the difference is due to the difference in measuring procedures.

No such measurement ever been made.....an SR observer predicts that an
observed clock runs slow by a factor of 1/gamma.

>
> As I have said before, one must consider "measured" in general, and not just
> when a human makes direct, quantitative measurements. For instance, this permits
> us to construct pion beamlines.
>
>         Charged pions are unstable particles with an intrinsic lifetime
>         of 26 nanoseconds. In a beamline they move with speed >0.99 c.

This is an assertion of the PoR that a moving pion will decay at 26 ns
according to a clock moving along with it. However, 26ns on the moving
pion clock is worth gamma*26 ns on the lab clock. What this mean is
that the moving pion has a life time of gamma*26 ns on the lab clock.
That's why a moving pion is able to travel a greater distance than the
lab pion before decaying. There is no absolute time dilation.

Ken Seto

kenseto

unread,
Dec 7, 2010, 5:36:12 PM12/7/10
to

No...if they start out from Buffalo simultaneously and arrive at
Chicago simultaneously the difference in the odometer readings is due
to the higher speed (wrt the road) of the car that traveled the longer
distance.

Ken Seto

Stamenin

unread,
Dec 7, 2010, 6:10:05 PM12/7/10
to
On Nov 26, 3:07 am, Shane williams <shane.2471...@gmail.com> wrote:
> Can anyone explain what lead Einstein to the theory of special
> relativity?  I know it was something to do with Maxwell's equations.
> I guess he had no way of testing or observing that time slows down the
> faster you go so what lead him to that conclusion?  Did it explain
> something that had been observed but there was no explanation for?
> How did he do it?
>
> TIA

The answer is simple, The Lorentz transformation did it. But the
Lorentz transformation is an errant mathematical relation and this is
the cause why the special relativity is errant too. LT is an
approximation of the Galilei Transformation.

harald

unread,
Dec 7, 2010, 6:38:13 PM12/7/10
to
On Dec 5, 4:29 am, Uncle Ben <b...@greenba.com> wrote:
> On Dec 4, 9:20 pm, Shane williams <shane.2471...@gmail.com> wrote:
>
>
>
> > On Nov 28, 5:25 am, Tom Roberts <tjroberts...@sbcglobal.net> wrote:

>
> > > Shane williams wrote:
> > > > Can anyone explain what lead Einstein to the theory of special
> > > > relativity?
>
> > >          discussed the significance of symmetry principles, which is the
> > >          key point.]
>

> > > Tom Roberts
>
> > Thanks.
>
> > So how much can a non-physicist (but with a science degree) expect to
> > understand about special relativity and the fact that time slows down
> > and length contracts the faster you go.  If I get a book about special
> > relativity from the library, can I expect it to explain why we think
> > that time slows down.
>
> > It seems to me that to get any idea of why we would expect that time
> > slows down and the speed of light is the same for all observers, you
> > need to understand Maxwell's equations and the things that lead
> > Einstein to the theory of special relativity.  Without understanding
> > that, all a book can do is explain what the postulates of special
> > relativity are and what the consequences of that are.
>
> > I have a long-standing desire to understand more about special
> > relativity but I'm thinking that unless I learn bout Maxwell's
> > equations and whatever lead Einstein to relativity, I'm not going to
> > understand why anyone would expect that time slows down the faster you
> > go, let alone how Einstein came up with E = MC*C- Hide quoted text -

>
> > - Show quoted text -
>
> Shane, none of us really understands relativity at an intuitive level,

Ben, why would you think that?
Best regards,
Harald

> but you can learn better than most what it says, at least.
>
> To my mind one of the most difficult things to learn is exactly what
> is meant by saying that speeding clocks run slow.  It may not quite be
> what you think.
>
> What many miss at first is that clocks can run at different rates at
> the same time.  I am purposely being a bit challenging when I put it
> this way, but I will explain.
>
> One could say that the altitude of a building can have different
> values simultaneously.  This is because you can choose different
> reference levels.  The altude referred to the street is one thing, and
> the altitude referred to mean sea level is another.  There is clearly
> no contradiction here.


>
> Consider a clock on a train moving through a station.  Special
> relativity says that the clock can have different rates at the same
> time: With respect to the train, the clock is motionless and has its
> maximum rate. With respect to the station, the same clock is moving
> and has a slower rate.
>
> This is quite strange, but experiments show it to be true. I don't
> believe that study of Maxwell's equations will help you much to
> believe it until you reach the level of sophistication of a Tom
> Roberts or Paul Draper (PD), but a review of the Hafely-Keating
> experiment (see wikipedia) may be helpful. They flew around the world
> with an atomic clock and saw that the clock gaiined or lost time
> according to which way they travelled: with or against the earth's
> rotation (east to west or west to east).
>
> Subsequent refinements of this experiment have confirmed time dilation
> to be quantitatively extremely close to what Einstein's theories
> predict, including a second effect due to Earth's gravity.
>

> Uncle Ben

Bruce Richmond

unread,
Dec 7, 2010, 10:55:09 PM12/7/10
to
On Dec 7, 9:15 am, jem <x...@xxx.invalid> wrote:
> Bruce Richmond wrote:
> > On Dec 6, 8:02 am, stevendaryl3...@yahoo.com (Daryl McCullough) wrote:
> >> Uncle Ben says...
>
> >>> On Dec 5, 8:47=A0pm, Tom Roberts<tjroberts...@sbcglobal.net>  wrote:
> >>> I agree that I am using the language a little differently than
> >>> convention dictates.  But you are too!
>
> >>> You emphasize that the varying rates or lengths are only MEASURED, not
> >>> intrinsic. No wonder the student's first reaction will be, "well, why
> >>> don't they measure it differently if the usual measurement is not
> >>> correct?"
>
> >> I think it is a mistake to think "Oh, if I had only used phrase X" or
> >> "Oh, if I had only not used phrase Y" then the student would not have
> >> become confused. Students become confused because it is a difficult
> >> subject.
>
> > It is not as difficult as some make it.  The phrasing used by Ben
> > confuses the issue.  A standard clock ticks at a rate of one second
> > per second in it's rest frame.
>
> No.  A standard clock doesn't tick at the rate of 1 second per second,
> which is meaningless.  It ticks at the rate of 1 second per *tick*
> (more generally, X seconds per tick).  IOW, for each specific
> recognizable, repetitive change (i.e. tick) in a standard clock, the
> clock registers 1 second (more generally, X seconds).

http://en.wikipedia.org/wiki/Second

"Since 1967, the second has been defined to be the duration of
9,192,631,770 periods of the radiation corresponding to the transition
between the two hyperfine levels of the ground state of the caesium
133 atom."

The clock is constructed to record seconds as defined by the standard
above. A standard clock at rest will tick at the same rate as all the
other standard clocks at rest in the same frame. If you use one of
those other clocks to measure its rate you will measure that it
records one second per second. Whether you want to consider it to
tick once per second, or 9,192,631,770 times per second is
irrelevant. The point is it measures seconds as defined by the
standard.

If you take that same clock and set it in motion the clocks still at
rest will measure it to tick slower. It will go out of sync with the
clocks that it previously stayed in sync with. But it will now stay
in sync with all the standard clocks at rest in its new rest frame.
It will be recording seconds in its new rest frame as defined in the
standard above. No adjustments had to be made to get it to tick at
the same rate as those other clocks, and they will measure that it
records one second per second, despite what its old frame measures.

I find it funny that you said "No" but what you described amounts to
the same thing I am saying.

> > It is measured to tick at different
> > rates in frames moving relative to it.
>
> No.  From any perspective whatsoever, the measurement of a clock's
> tick rate (i.e., the amount of time the clock registers between each
> tick it makes), will yield the same result.

If we are in a frame that observes that clock to be moving we will
measure it to be running slow. It will still be recording one second
for every 9,192,631,770 transitions of the caesium 133 atom, but the
caesium 133 atom will be measured to make those transitions slower
because it is in motion relative to us. That is why the link above
includes, "This definition refers to a caesium atom at rest". That is
why the intrinsic rate of a standard clock is one second per second
*in its rest frame*. When it is moving in a frame it is not a
standard clock in that frame.

Of course observers at rest with respect to this moving clock have
every right to consider themselves at rest. And for them the clock
does fulfill the definition above. And when they use their clocks,
synchronized in their frame, to measure the rate of the clocks in the
other frame, they will find them running slow due to relative
simultaneity.

> >  Phrases such as "moving clocks
> > run slow" imply that the clock is somehow different, when in fact you
> > measure it to run slow
>
> No.  Only those who don't know how to measure a clock's tick rate will
> measure a standard clock to run slow (or fast).

Observers A and B are at rest with respect to each other. A moving
clock passes A, who records the time on the passing clock and the time
on his own clock. When the moving clock passes B he does the same.
Subtract the times recorded by A from the times recorded by B to
calculate the elapsed times. The moving clock will have accumulated
less elapsed time then was calculated to have elapsed on the clocks of
A and B. Of course the synchronization of the clocks at A and B
affect the calculation.

> > because you are using a different coordinate
> > system  in which the clocks making the measurement are synchronized
> > differently.
>
> No.  It's because you're not measuring a clock's tick rate when you
> determine the amount of time that elapses on *your* clock per tick of
> some other clock (anymore than you'd be measuring the rate of someone
> else's odometer by determining the ratio of the distance you travel
> per tick of their odometer).

When we measure things we use our own coordinates to make the
measurements. If I am standing on the side of the road I measure the
tree next to me to be moving at zero mph. The observer driving by may
measure it to be traveling at 30 mph relative to him, but to me it is
not moving. Likewise I measure the car to be traveling at 30 mph,
while the observer in the car considers it to be stationary. When we
measure the rate of a moving clock we use our stationary clocks. To
do otherwise is called frame jumping.

Reading your statment above again I realize you may have been trying
to say that our measurement of the rate that the clock is ticking is
not the tick rate of the clock, which would be its intrinsic tick
rate. I would agree with that, but it is phrasing like that which you
used that make things difficult. Using "intrinsic tick rate" vs
"measured tick rate" is much less confusing than "tick rate" vs "rate
of tick".

> > The key to understanding SR is  to keep in mind that each
> > frame uses its own coordinate system to make measurements, and because
> > of how those coordinate systems are set up you must use the Lorentz
> > transformation to translate between the coordinate systems.
>
> One key to understanding SR is to avoid thinking that you know the key
> to understanding it, before you've understood it.

I understand it quite well thank you.

> General comment:
>
> The motivation here isn't pedantic stubbornness - the failure to
> *emphasize* the distinction between "a clock's tick rate" and "a rate
> at which a clock ticks", is a proven pedagogical faux pas that's
> responsible for many deep-seated misconceptions,

Which is exactly the concept Tom Roberts was trying to get across to
Ben. The clock's intrinsic tick rate is one second per second in it's
rest frame. The rate at which it is measured to tick in other frames
will differ. That wording is much easier for normal people to
understand than your "tick rate" vs "rate of tick". Your wording is
exactly the type of thing I was telling Daryl makes things more
difficult than they need to be.

> chief among which is
> the failure to understand that, in the Einsteinian model, time
> differences between events are a consequence of route-dependent time,

> not differentially ticking clocks.- Hide quoted text -

Uncle Ben

unread,
Dec 7, 2010, 11:02:11 PM12/7/10
to
On Dec 7, 6:38 pm, harald <h...@swissonline.ch> wrote:
> On Dec 5, 4:29 am, Uncle Ben <b...@greenba.com> wrote:
>
>...>

> > Shane, none of us really understands relativity at an intuitive level,
>
> Ben, why would you think that?
> Best regards,
> Harald
>
Well, maybe you have better intuition than I, Harald, but I find it
highly non-intuitive to say that c is invariant under the LT. I have
learned that it is true, but I have to ignore intuition to deal with
it. So I just do the math. The same with time dilation.

SRT was resisted so strongly by physicists that I have to believe that
I am not alone. But maybe I over-stated it.

Cheers!

Ben

Uncle Ben

unread,
Dec 7, 2010, 11:24:16 PM12/7/10
to
> > not differentially ticking clocks.- Hide quoted text -- Hide quoted text -
>
> - Show quoted text -- Hide quoted text -

>
> - Show quoted text -

Bruce, I hope you realize that Tom Roberts and I differ about
pedagogy, not physics. He doesn't like me to shock the student by
referring to clocks having different rates according to their speeds;
and I don't like his talk about measurement as if it doesn't yield
truth. Ultimately in discussion in a class, we would come out at the
same understanding IMHO.

I think the history of physicists flubbing Bell's spaceship problem en
masse supports my position.

Ben

Tom Roberts

unread,
Dec 8, 2010, 12:46:58 AM12/8/10
to
Uncle Ben wrote:
> The quibble that Tom and I disagree over is whether to stop right
> there or to add that when you measure the rate of the same clock WITH
> RESPECT TO A DIFFERENT FRAME OF REFERENCE, one can say that the clock
> has a different rate.

The only rate a clock _HAS_ it its own rate. What you attempt to call the
"clock's rate" is not owned or possessed BY THE CLOCK. It is, rather, a
relationship between the clock and the other frame. This _IS_ how these words
are used in English.


Tom Roberts

Tom Roberts

unread,
Dec 8, 2010, 1:08:30 AM12/8/10
to
PD wrote:
> Yes, if it's the REST frame value that you're going after, agreed.
> The electron's *rest* mass or *invariant* mass is 0.511 MeV.

Sure. But the nomenclature today is that mass is invariant, so your emphasized
adjectives are redundant: the electron's mass is 0.511 MeV/c^2. Yes, this value
is invariant, and is therefore the value in its rest frame.

Those adjectives were needed in olden days when the nomenclature
was evolving in this area; no longer.

Anecdotal evidence: a few years ago, in three separate meetings at Fermilab I
was able to ask each group "Who thinks mass varies with velocity?" -- NOBODY
replied in the affirmative, and several people wondered aloud how I could ask
such a silly question. Two of these meetings were collaboration meetings of
experiments in high energy physics, and one was a seminar on accelerator physics
-- precisely the people who use SR in their everyday work. There were over 100
Ph.D. physicists in these three meetings; I knew most of the other people by
name, and they all knew me from my then-recent colloquium on the experimental
basis of SR.


Tom Roberts

Androcles

unread,
Dec 8, 2010, 1:16:36 AM12/8/10
to

"Uncle Ben" <b...@greenba.com> wrote in message
news:07720d9c-cde9-46ed...@n10g2000yqd.googlegroups.com...

On Dec 7, 6:38 pm, harald <h...@swissonline.ch> wrote:
> On Dec 5, 4:29 am, Uncle Ben <b...@greenba.com> wrote:
>
>...>
> > Shane, none of us really understands relativity at an intuitive level,
>
> Ben, why would you think that?
> Best regards,
> Harald
>
Well, maybe you have better intuition than I, Harald, but I find it
highly non-intuitive to say that c is invariant under the LT. I have
learned that it is true, but I have to ignore intuition to deal with
it. So I just do the math.


==========================================
Bwahahahahahahahaha!

1) But it is not possible without further ASSUMPTION to compare, in respect
of time, an event at A with an event at B.
2) We ASSUME that this definition of synchronism is free from
contradictions, and possible for any number of points;
3) In agreement with experience we further ASSUME the quantity 2AB/(t'A-tA)
= c
4) Current kinematics tacitly ASSUMES that the lengths determined by these
two operations are precisely equal
5) and where for brevity it is ASSUMED that at the origin of k, tau = 0,
when t=0.
6) If no ASSUMPTION whatever be made as to the initial position of the
moving system and as to the zero point of tau
7) We now have to prove that any ray of light, measured in the moving
system, is propagated with the velocity c, if, as we have ASSUMED, this is
the case in the stationary system
8) If we ASSUME that the result proved for a polygonal line is also valid
for a continuously curved line,
9) and our equations ASSUME the form
10) When phi = 0 the equation ASSUMES the perspicuous form
11) the equation for phi' ASSUMES the form
12) for the law of motion of which we ASSUME as follows
13) we may and will ASSUME that the electron, at the moment when we give it
our attention
14) From the above ASSUMPTION, in combination with the principle of
relativity

Doing the math:

--
r_AB/(c+v) = r_AB/(c-v). References given:
<http://www.fourmilab.ch/etexts/einstein/specrel/www/figures/img6.gif>
<http://www.fourmilab.ch/etexts/einstein/specrel/www/figures/img11.gif>

Let r_AB = 480 million metres,
let c = 300 million metres/sec,
let v = 180 million metres/sec.

480/(300-180) = 480/(300 +180)
480/(120) = 480/(480)
4 = 1

"In agreement with experience we further assume the quantity
2AB/(t'A-tA) = c to be a universal constant, the velocity of
light in empty space." --§ 1. Definition of Simultaneity --
ON THE ELECTRODYNAMICS OF MOVING BODIES By A. Einstein

"the velocity of light in our theory plays the part, physically, of an
infinitely great velocity"--§ 4. Physical Meaning of the Equations
Obtained in Respect to Moving Rigid Bodies and Moving Clocks
--ON THE ELECTRODYNAMICS OF MOVING BODIES By A. Einstein

In agreement with experience we further assume four seconds plays the
part, physically, of one second, the idiocy of raving lunatics in
Relativityland.

shuba

unread,
Dec 8, 2010, 1:37:19 AM12/8/10
to
Uncle Ben wrote:

> There exist relativistic stresses in constrained solids subjected to
> acceleration (in addition to the classically understood compression
> that propagates the acceleration).

There are minor, yet important inaccuracies in that statement. For one,
relativistic stresses are not *in addition* to nonrelativistic ones. That
kind of sloppy presentation is precisely what students don't need. Also,
there is no reason to use the word "constrained". Acceleration of solids
does change stress, and this should be stated simply. Strictly speaking,
it's also true for liquids and plasmas, an interesting and perhaps non-
obvious fact that might be worth mentioning.

A student wishing to understand relativity would be well advised to
consider why nonrelativistic stress is desrcibed by a 3x3 tensor in
space. Then the reason relativistic stress is desrcibed by a 4x4 tensor
is less mysterious, being precisely the same reason that momentum is
relativisticly defined as a four-vector -- it obeys the symmetry of
boosts as well as spatial rotations.


---Tim Shuba---

Bruce Richmond

unread,
Dec 8, 2010, 7:35:22 AM12/8/10
to

I realize that you agree on the physics and I agree with Tom. I think
you do your students a disservice by confusing them in your attempt to
make your point. It is very similar to the old discussions about
mass. It was very confusing when it used to be said that mass
increases with speed when what was meant was relativistic mass. Of
course those that *knew* SR knew the difference. But if you didn't
already know SR it was just one more stumbling block in the way of
understanding.

> I think the history of physicists flubbing Bell's spaceship problem en
> masse supports my position.
>

> Ben- Hide quoted text -

kenseto

unread,
Dec 8, 2010, 8:53:41 AM12/8/10
to
On Dec 7, 10:55 pm, Bruce Richmond <bsr3...@my-deja.com> wrote:

The problem with this definition is that it assumes that a period of
Cs 133 radiation represents the same *duration* in all frames of
reference. It does not. The GPS illustrates this clearly. The GPS
second is redefined to have 9,192,631,774.25 periods of Cs 133
radiation....the added 4.25 periods of Cs 133 radiation is designed to
make the GPS second has the same *duration* as the ground clock
second. Except for the daily drift, the redefined GPS second make the
GPS in synch with the ground clock permanently.
BTW that's why different observers predict the same clock *A* runs at
different rates....the reason is that different observers' clock
second has different *duration*.

Ken Seto

> > not differentially ticking clocks.- Hide quoted text -- Hide quoted text -
>
> - Show quoted text -- Hide quoted text -

PD

unread,
Dec 8, 2010, 9:00:39 AM12/8/10
to

There is no such assumption, Ken, other than the one that you add to
what is written above.
There is no statement that the second, so defined, will be an absolute
duration of any kind.
The standard applies to the *local, comoving* reference frame only.
If you move to a different reference frame, then the oscillations of
that particular cesium atom will no longer serve as the standard of a
second any longer, because you are no longer in the local, comoving
frame.
However, if you choose a *different* cesium atom, one that is at rest
in your reference frame, then *this* atom will serve as the standard
of the second.
In this way, it is a common standard, applicable to any inertial
reference frame.

PD

unread,
Dec 8, 2010, 9:06:02 AM12/8/10
to

Agreed, but that is a different audience than what you have here.
Here you have an audience who is still reading materials that are less
sanitized, and so some care must be taken.

On a side front, I'll wager that if you ask the same question to a
group of incoming freshman, you'll get a remarkably different survey
response. This is because whatever contact they've gotten to
relativity up until that point is at the very cursory or lay-market
level, and that is *remarkably* polluted with these antiquated notions
you don't think people have to even deal with anymore. That means that
before you can assume people are trained on the meaning of things,
someone will have had to spend some time *untraining* them in the
improper meaning of things.

jem

unread,
Dec 8, 2010, 9:06:54 AM12/8/10
to

A standard clock's tick rate is (and will be correctly measured to be)
X seconds per tick (where X depends on calibration) from EVERY
perspective (i.e., in EVERY frame).

A clock is a measuring device, as is a speedometer. What are you
going to measure for the rate of the speedometer in a passing car?

That wording is much easier for normal people to
> understand than your "tick rate" vs "rate of tick". Your wording is
> exactly the type of thing I was telling Daryl makes things more
> difficult than they need to be.
>

ANY wording that indicates that the tick rates of Relativity's
standard clocks can vary is simply counter-factual. The fact that the
subject's literature is full of loosely-worded statements to the
contrary, has done nothing but add to the difficulty of acquiring a
clear understanding of the Relativistic model.

"Relativistic mass" created enough confusion to be discarded, and that
term even had a special adjective to differentiate it from the primary
quantity of interest. References that imply that the ratio, ticks on
one clock per time on another, is a clock's tick rate, need to be
discarded for the same reason.

kenseto

unread,
Dec 8, 2010, 9:11:02 AM12/8/10
to

If not then why do you compare a traveling clock second with a stay at
home clock second directly to reach the conclusion that the traveling
clock is younger? Are you just making things up as you go along?

Ken Seto

> The standard applies to the *local, comoving* reference frame only.
> If you move to a different reference frame, then the oscillations of
> that particular cesium atom will no longer serve as the standard of a
> second any longer, because you are no longer in the local, comoving
> frame.
> However, if you choose a *different* cesium atom, one that is at rest
> in your reference frame, then *this* atom will serve as the standard
> of the second.
> In this way, it is a common standard, applicable to any inertial
> reference frame.
>
>
>
> > It does not. The GPS illustrates this clearly. The GPS
> > second is redefined to have 9,192,631,774.25 periods of Cs 133
> > radiation....the added 4.25 periods of Cs 133 radiation is designed to
> > make the GPS second has the same *duration* as the ground clock
> > second. Except for the daily drift, the redefined GPS second make the
> > GPS in synch with the ground clock permanently.
> > BTW that's why different observers predict the same clock *A* runs at
> > different rates....the reason is that different observers' clock
> > second has different *duration*.
>

> > Ken Seto- Hide quoted text -

jem

unread,
Dec 8, 2010, 9:15:02 AM12/8/10
to
Tom Roberts wrote:
> PD wrote:
>> Yes, if it's the REST frame value that you're going after, agreed.
>> The electron's *rest* mass or *invariant* mass is 0.511 MeV.
>
> Sure. But the nomenclature today is that mass is invariant, so your
> emphasized adjectives are redundant: the electron's mass is 0.511
> MeV/c^2. Yes, this value is invariant, and is therefore the value in its
> rest frame.
>
> Those adjectives were needed in olden days when the nomenclature
> was evolving in this area; no longer.
>
> Anecdotal evidence: a few years ago, in three separate meetings at
> Fermilab I was able to ask each group "Who thinks mass varies with
> velocity?"

Next time ask "Who thinks length varies with velocity?".

PD

unread,
Dec 8, 2010, 9:41:31 AM12/8/10
to

Because in science, Ken, "comparison" means simply taking the readings
from two instruments and seeing which one is larger, if any. YOU, on
the other hand, have placed the ADDITIONAL imposition that two
readings cannot be compared at all unless they are calibrated against
an absolute standard, where in your mind (and YOUR MIND ONLY)
"absolute" means yielding the same results across all frames. But
that's just you.

Scientists compare those values using the meaning of "comparison" I've
just described. You say that isn't a comparison and for them to use
the word "comparison" an absolute standard is implied. It's not.
That's just you.

kenseto

unread,
Dec 8, 2010, 9:52:46 AM12/8/10
to

No idiot....when you compare the traveling clock second with the stay
at home clock second directly you are assumiong that they have the
same duration.....

PD

unread,
Dec 8, 2010, 10:35:35 AM12/8/10
to

Absolute duration? No, I'm not making any such assumption. You may be,
but scientists don't, no.

PD

unread,
Dec 8, 2010, 10:42:16 AM12/8/10
to
On Dec 8, 8:15 am, jem <x...@xxx.invalid> wrote:
> Tom Roberts wrote:
> > PD wrote:
> >> Yes, if it's the REST frame value that you're going after, agreed.
> >> The electron's *rest* mass or *invariant* mass is 0.511 MeV.
>
> > Sure. But the nomenclature today is that mass is invariant, so your
> > emphasized adjectives are redundant: the electron's mass is 0.511
> > MeV/c^2. Yes, this value is invariant, and is therefore the value in its
> > rest frame.
>
> > Those adjectives were needed in olden days when the nomenclature
> > was evolving in this area; no longer.
>
> > Anecdotal evidence: a few years ago, in three separate meetings at
> > Fermilab I was able to ask each group "Who thinks mass varies with
> > velocity?"
>
> Next time ask "Who thinks length varies with velocity?".
>
>   -- NOBODY replied in the affirmative, and several people

Interestingly, I bet you get a number in between if you ask the same
question about particle lifetime.

kenseto

unread,
Dec 8, 2010, 11:53:01 AM12/8/10
to

Hey idiot I didn't say absolute duration. I said the same duration.


> No, I'm not making any such assumption. You may be,
> but scientists don't, no.

Yes you were making assumption....that's why you compare a traveling
clock second with a stay at home clock second directly.
You are not a scientist. You are a SR religous fanatic.

Ken Seto

PD

unread,
Dec 8, 2010, 12:37:17 PM12/8/10
to

I don't know what you mean by the term then.
In an inertially moving clock, the second is defined the same way --
it is a certain number of oscillations in a transition of the cesium
atom local to the clock -- each and every time.

You make the claim that the second is DEFINED differently for the GPS
clocks, but as has been pointed out to you at least a hundred times,
this is not what is actually done. What is actually done is documented
publicly on the web.

>
> > No, I'm not making any such assumption. You may be,
> > but scientists don't, no.
>
> Yes you were making assumption....that's why you compare a traveling
> clock second with a stay at home clock second directly.

No sir. A comparison only involves what I told you it does. There is
NO assumption about a comparison standard involved.

> You are not a scientist. You are a SR religous fanatic.

What do you think a scientist is, exactly, Ken?

kenseto

unread,
Dec 8, 2010, 5:48:02 PM12/8/10
to

In order to compare clock second in different frames directly you have
to assume that each second represent the same duration. But in real
life a cock second in different frame does not represent the same
duration and thus you can'r compare a traveling clock second with a


stay at home clock second directly.

> In an inertially moving clock, the second is defined the same way --
> it is a certain number of oscillations in a transition of the cesium
> atom local to the clock -- each and every time.

A period of Cs 133 radiation does not represent the same duration in
different inertial frames. What this mean is that the definition for a
clock second does not represent the same duration in different
frames.....therefore you can't compare clock seconds in different
frames directly. You must convert the traveling clock seconds into the
value on the stay at home clokc before making the comparison.

Bruce Richmond

unread,
Dec 8, 2010, 8:55:20 PM12/8/10
to

No, it is only measured to tick at X seconds per tick in its own rest
frame. In all inertial frames that are moving relative to its rest
frame it will increment one second for every 9,192,631,770 transitions
of the clock's caesium 133 atom, but the moving caesium 133 atom will
be measured to transition slower than the caesium 133 atom in our
standard clock due to its motion.

We have arranged a number of standard clocks along the side of some
train tracks and synchronized them with each other. A clock on a
moving train happens to coincide with the track clock at x=0 when both
clocks read 0. The train clock passes the track observer at x=5 at
the time t=8. What time does the track observer see on the train
clock? Plug the info into the Lorentz transformation and find out if
you like. It will give you an answer less than 5. That is the
elapsed time on the train clock as measured by the track observers.
They measured the train clock to be running slow.

> A clock is a measuring device, as is a speedometer.  What are you
> going to measure for the rate of the speedometer in a passing car?

If I am in a moving car when you go by I will see it reading your
speed relative to the road, not to me. Are you trying to invoke an
absolute frame like Ken?

> > That wording is much easier for normal people to
> > understand than your "tick rate" vs "rate of tick".   Your wording is
> > exactly the type of thing I was telling Daryl makes things more
> > difficult than they need to be.
>
> ANY wording that indicates that the tick rates of Relativity's
> standard clocks can vary is simply counter-factual.

Not if you make it clear that you are talking about the *measured*
tick rate as opposed to the *intrinsic* tick rate. The intrinsic rate
is not observer dependent while the measured rate is.

> The fact that the
> subject's literature is full of loosely-worded statements to the
> contrary, has done nothing but add to the difficulty of acquiring a
> clear understanding of the Relativistic model.
>
> "Relativistic mass" created enough confusion to be discarded, and that
> term even had a special adjective to differentiate it from the primary
> quantity of interest.  

Relativistic mass was confusing because the word "relativistic" often
got dropped.

> References that imply that the ratio, ticks on
> one clock per time on another, is a clock's tick rate, need to be
> discarded for the same reason.

Lots of luck with that.

>
>
> >> chief among which is
> >> the failure to understand that, in the Einsteinian model, time
> >> differences between events are a consequence of route-dependent time,

> >> not differentially ticking clocks.- Hide quoted text -- Hide quoted text -

Tom Roberts

unread,
Dec 9, 2010, 12:16:32 AM12/9/10
to
Uncle Ben wrote:
> You emphasize that the varying rates or lengths are only MEASURED, not
> intrinsic. No wondner the student's first reaction will be, "well, why

> don't they measure it differently if the usual measurement is not
> correct?"
> There is no-one that we know of in distant galaxies doing any
> measurement. Yet we presume that relativistic effects occur there as
> well as here.

You obviously missed the places where I have said that "measurement" must be
taken generally, and not just for humans making quantitative observations.

And you also missed where I said this is a RELATIONSHIP between the clock and
the frame used for the observation of the clock's tick rate. That's obviously
more than just a measurement artifact.

Is carrying a ladder through a doorway a "measurement" of the
ladder's length? Or of the ladder's length projected onto the
plane of the doorway? -- NO. But yet, the relationship between
ladder and doorway determines whether or not it gets through.
Similarly, such relationships govern physical properties in
relativity, where relative velocity is an analogous rotation....


> To put it another way, we all accept the uncertainty principle in QM.
> Measurement does affect what is being measured. The way you put
> things is that there is a similar principle at work in relativity.

Not at all. I have no idea where you got that from. I have not mentioned QM or
the uncertainty principle in this thread at all.

Bottom line: the result of a measurement depends on the way that measurement is
performed. This OUGHT to be obvious. One of the consequences is that if you
measure the tick rate of a moving clock, the procedure you use will NECESSARILY
differ from the procedure you would use for a clock at rest, so it is not a
priori obvious they should yield the same results; in practice, of course, they
don't.

And this is QUITE DIFFERENT from the Heisenberg uncertainty
principle of QM. This is purely classical.


> Measurement cannot be relied upon to give the truth.

This is correct, but for a VERY different reason than you suppose -- there is no
"truth" in science.


> I say that you are changing the meaning of "measurement" when you
> distinguish between what is measured and what is true.

I also have never mentioned or discussed "what is true" Indeed, I have
repeatedly pointed out that in science there is no "what is true", because we
can never achieve certainty about the world. There are only theories that are
VALID in some domain, there are NEVER "true" theories.

I have not changed the meaning of measurement. But I do insist that one does not
ascribe to a given measurement aspects that it does not have. Measuring the time
interval between ticks of a moving clock does NOT measure that clock's tick
rate; it measures the clock's tick rate PROJECTED ONTO THE FRAME USED FOR THE
MEASUREMENT, which is a different quantity altogether.

In Newtonian mechanics there was no need to distinguish between
frames for such measurements, and no such qualification was needed.
In relativity there is, and it is ESSENTIAL to mention the frame
when discussing such measurements -- your insistence on not
doing so is just plain wrong.


A geometric analogy is appropriate: consider a meterstick -- it is 1 meter long.
Now if you were to measure the distance between its endpoints projected onto the
floor while holding the meterstick at 45 degrees from vertical, would you say "I
just measured the length of the meterstick to be 0.707 meters"? -- I doubt you
would consider that a reasonable statement, yet that is PRECISELY WHAT YOU ARE
DOING. You should of course call that a measurement "of the meterstick's length
projected onto the floor" -- it is CLEARLY not a measurement of its length.


> I prefer to
> say that physicists making measurements are discovering what is true:

I prefer to do and discuss science, not whatever it is you are attempting to do.


> that the lifetime of a cosmic-ray muon IS what is it MEASURED to be.

But those words do not accurately reflect this: the lifetime of a cosmic-ray
muon, OR ANY OTHER MUON, is 2.2 microseconds. This is so for all muons,
regardless of which frame they might be at rest in. An observer on earth
measuring a cosmic-ray muon is not at rest in the muon's rest frame, and so
obtains a different value.

Stated differently: the lifetime of an unstable particle is an intrinsic
property of the particle. That implies it is invariant, and is not subject to
change by looking at it from a frame in which it is moving. This is just like
mass....


> And if the answer depends on its speed w.r.t. the laboratory, get used
> to it!

But the answer of such a measurement is most definitely not "the lifetime of the
muon". It is the lifetime of the muon projected onto the lab frame used for
measurement.

[This is getting overly repetitive; don't expect me to continue.]


Tom Roberts

Tom Roberts

unread,
Dec 9, 2010, 12:29:12 AM12/9/10
to
PD wrote:
> On Dec 8, 8:15 am, jem <x...@xxx.invalid> wrote:
>> Tom Roberts wrote:
>>> PD wrote:
>>>> Yes, if it's the REST frame value that you're going after, agreed.
>>>> The electron's *rest* mass or *invariant* mass is 0.511 MeV.
>>> Sure. But the nomenclature today is that mass is invariant, so your
>>> emphasized adjectives are redundant: the electron's mass is 0.511
>>> MeV/c^2. Yes, this value is invariant, and is therefore the value in its
>>> rest frame.
>>> Those adjectives were needed in olden days when the nomenclature
>>> was evolving in this area; no longer.
>>> Anecdotal evidence: a few years ago, in three separate meetings at
>>> Fermilab I was able to ask each group "Who thinks mass varies with
>>> velocity?"
>> Next time ask "Who thinks length varies with velocity?".
>>
>> -- NOBODY replied in the affirmative, and several people
>
> Interestingly, I bet you get a number in between if you ask the same
> question about particle lifetime.

I very much doubt it. Lifetime is CLEARLY an intrinsic property of the muon,
just like mass. Our reference books all give masses and lifetimes for muons, and
they don't bother to mention anything at all about which frame they apply to,
because it is OBVIOUS they are intrinsic properties. Ditto for charge, magnetic
moment, electric dipole, and the discrete quantum numbers of the muon (lepton
numbers, etc.).

Similarly, when we say "this meterstick is 1 meter long", we don't bother to
mention which frame that applies to, because it is obvious we are discussing an
intrinsic property of the meterstick.

Anecdotally, I don't ever recall anyone discussing "lifetime of the muon in the
lab", nor "length of the muon in the lab".


Tom Roberts

Uncle Ben

unread,
Dec 9, 2010, 1:12:46 AM12/9/10
to
> Tom Roberts- Hide quoted text -

>
> - Show quoted text -

I am perfectly willing to let you have the last word on this matter,
except for an observation no-one has mentioned before.

You are a sophicated student of relativity; for you the idea of a
measurement projected onto a frame of reference may be very clear and
precise. I have been the teacher of beginning students of relativity,
to whom that idea conveys nothing.

Teachers of relativity for many decades have emphasized as you do that
measurements of things, being frame-dependent, should be described as
such: just measurements. They even use the term "appears"; the moving
meter-stick "appears" to be shorter than a meter.

Beginning students wonder why we say that. We certainly do not say
that in other branches of physics. If we measure a temperature, we
say "the temperature is 27 deg," not "the temperature appears to be 27
deg." In the latter case we seem to be hedging our claims, as if we
do not really believe them. There must be illusions at work.

That is why, IMHO, so many top physicists guessed wrong when
confronted with the Bell spaceship problem. They could not say that
the string just "appears" to break -- that would be ridiculous -- so
they concluded that it would not break.

Specialists in relativity got it right, of course. Those others who
got it wrong could be persuaded by the use of a co-moving frame and an
appeal to the relativity of simultenaity.

But those few others who got it right immediately had the idea that
when we measure a contraction as per Lorentz, the string is "really"
contracted if free to be so, and the constraint creates a stress that
breaks the string. I believe this to be a better way to teach in the
first place. Of course one describes the results as frame-dependent;
but that is a much easier concept to teach beginners than "projections
onto a frame of reference."

When I tell beginners that a clock can have many rates, that is the
beginning of the conversation, not the end of it. It is not over
under it is made clear that the lack of uniqueness of rate is because
there is another variable relevant in relativity that is not so in
Newtonian physics: the speed of the clock, which depends on the frame
of reference. I contend that this is much more meaningful to beginning
students than the "projection of measurement."

Uncle Ben


Androcles

unread,
Dec 9, 2010, 1:20:10 AM12/9/10
to

"Uncle Ben" <b...@greenba.com> wrote in message
news:17fb8332-640c-44eb...@l17g2000yqe.googlegroups.com...

Uncle Ben
=============================================
Taking money under false pretenses is FRAUD. You should be sued
and jailed.

kenseto

unread,
Dec 9, 2010, 8:02:06 AM12/9/10
to

So does this mean that cesium clocks in relative motion run at
different intrinsic rates?

> We have arranged a number of standard clocks along the side of some
> train tracks and synchronized them with each other.  A clock on a
> moving train happens to coincide with the track clock at x=0 when both
> clocks read 0.  The train clock passes the track observer at x=5 at
> the time t=8.  What time does the track observer see on the train
> clock?  Plug the info into the Lorentz transformation and find out if
> you like.  It will give you an answer less than 5.  That is the
> elapsed time on the train clock as measured by the track observers.
> They measured the train clock to be running slow.

That's the problem....the LT only cover one possibility....that an
observed clock runs slow. The other possibility is that an observed
clock can run faster than the observer's clock. I suggest that you
read the paper in the following link:
http://www.modelmechanics.org/2008irt.dtg.pdf
BTW you said "It will give you an answer less than 5." Dont you mean
that "It will give you an answer less than 8."

Ken Seto

>
> > A clock is a measuring device, as is a speedometer.  What are you
> > going to measure for the rate of the speedometer in a passing car?
>
> If I am in a moving car when you go by I will see it reading your
> speed relative to the road, not to me.  Are you trying to invoke an
> absolute frame like Ken?
>
> > > That wording is much easier for normal people to
> > > understand than your "tick rate" vs "rate of tick".   Your wording is
> > > exactly the type of thing I was telling Daryl makes things more
> > > difficult than they need to be.
>
> > ANY wording that indicates that the tick rates of Relativity's
> > standard clocks can vary is simply counter-factual.
>
> Not if you make it clear that you are talking about the *measured*
> tick rate as opposed to the *intrinsic* tick rate.  The intrinsic rate
> is not observer dependent while the measured rate is.
>
> > The fact that the
> > subject's literature is full of loosely-worded statements to the
> > contrary, has done nothing but add to the difficulty of acquiring a
> > clear understanding of the Relativistic model.
>
> > "Relativistic mass" created enough confusion to be discarded, and that
> > term even had a special adjective to differentiate it from the primary
> > quantity of interest.  
>

> Relativistic mass was confusing because the word- Hide quoted text -
>
> - Show quoted text -...
>
> read more »

jem

unread,
Dec 9, 2010, 9:04:06 AM12/9/10
to
PD wrote:
> On Dec 8, 8:15 am, jem<x...@xxx.invalid> wrote:
>> Tom Roberts wrote:
>>> PD wrote:
>>>> Yes, if it's the REST frame value that you're going after, agreed.
>>>> The electron's *rest* mass or *invariant* mass is 0.511 MeV.
>>
>>> Sure. But the nomenclature today is that mass is invariant, so your
>>> emphasized adjectives are redundant: the electron's mass is 0.511
>>> MeV/c^2. Yes, this value is invariant, and is therefore the value in its
>>> rest frame.
>>
>>> Those adjectives were needed in olden days when the nomenclature
>>> was evolving in this area; no longer.
>>
>>> Anecdotal evidence: a few years ago, in three separate meetings at
>>> Fermilab I was able to ask each group "Who thinks mass varies with
>>> velocity?"
>>
>> Next time ask "Who thinks length varies with velocity?".
>>
>> -- NOBODY replied in the affirmative, and several people
>
> Interestingly, I bet you get a number in between if you ask the same
> question about particle lifetime.
>

I think a mixed response to that question would be appropriate, PD,
given that "particle lifetime" could refer to a property of the
particle (and so be velocity-independent), or to the observed duration
of an aging process (and so be velocity dependent in Relativity)
However, in the case of "length", I'm pretty sure the only unraised
hands in the room would belong to Roberts and those who didn't hear
the question.

jem

unread,
Dec 9, 2010, 9:25:58 AM12/9/10
to

No, they didn't. They measured the train clock to be accumulating time
at a slower rate than their own clocks, but as has been carefully
explained to you, in the Relativistic model, that doesn't constitute a
difference in the tick rates of the clocks.

>> A clock is a measuring device, as is a speedometer. What are you
>> going to measure for the rate of the speedometer in a passing car?
>
> If I am in a moving car when you go by I will see it reading your
> speed relative to the road, not to me.

You missed the simple point, Richmond, you'll measure the rate of the
speedometer to be the same as everybody else measures it, namely the
rate it registers. The speedometer's rate (and likewise the rate of
the driver's wristwatch) has nothing whatsoever to do with the
circumstances of anyone observing the speedometer (or the driver's
wristwatch).

Are you trying to invoke an
> absolute frame like Ken?
>
>>> That wording is much easier for normal people to
>>> understand than your "tick rate" vs "rate of tick". Your wording is
>>> exactly the type of thing I was telling Daryl makes things more
>>> difficult than they need to be.
>>
>> ANY wording that indicates that the tick rates of Relativity's
>> standard clocks can vary is simply counter-factual.
>
> Not if you make it clear that you are talking about the *measured*
> tick rate as opposed to the *intrinsic* tick rate. The intrinsic rate
> is not observer dependent while the measured rate is.
>

Until it sinks in that the measurement you're talking about is NOT a
measurement of a clock's tick rate, you're stuck with yet another
misconception about Relativity, Richmond. In the Relativistic model,
(ideal) clock tick rates are presumed NOT to change, so clearly, those
tick rates can't be measured to change without falsifying the model.

>> The fact that the
>> subject's literature is full of loosely-worded statements to the
>> contrary, has done nothing but add to the difficulty of acquiring a
>> clear understanding of the Relativistic model.
>>
>> "Relativistic mass" created enough confusion to be discarded, and that
>> term even had a special adjective to differentiate it from the primary
>> quantity of interest.
>
> Relativistic mass was confusing because the word "relativistic" often
> got dropped.
>
>> References that imply that the ratio, ticks on
>> one clock per time on another, is a clock's tick rate, need to be
>> discarded for the same reason.
>
> Lots of luck with that.
>

It's not me who needs the luck, Richmond, the price for that "moving
clocks run slow" sound bite is no longer being paid by me, only by
newcomers to the subject, and those unable to get past the deep-seated
misconceptions it's engendered.

PD

unread,
Dec 9, 2010, 10:46:04 AM12/9/10
to

No, you do not. That is a claim that YOU make, and you INSIST that it
must be true for everyone because you think it makes sense. But Ken,
science is not beholden to what you think makes sense.
So give it up.

> But in real
> life a cock second in different frame does not represent the same
> duration and thus you can'r compare a traveling clock second with a
> stay at home clock second directly.
>
> > In an inertially moving clock, the second is defined the same way --
> > it is a certain number of oscillations in a transition of the cesium
> > atom local to the clock -- each and every time.
>
> A period of Cs 133 radiation does not represent the same duration in
> different inertial frames.

But it means the same duration in the local frame where that cesium
atom is. And that's what the standard specifies and no more.

> What this mean is that the definition for a
> clock second does not represent the same duration in different
> frames.....therefore you can't compare clock seconds in different
> frames directly.

Yes, you can, because insistence on the same duration across different
inertial frames is not required for comparison, despite your fussy
demand that it should.

PD

unread,
Dec 9, 2010, 10:48:45 AM12/9/10
to

Anecdotally, that's not my experience. There's an interesting mixture,
depending primarily on the experimental context.

>
> Tom Roberts

It is loading more messages.
0 new messages